S1M1

¡Supera tus tareas y exámenes ahora con Quizwiz!

The hypothetical "stimulin" gene contains two exons that encode a protein of 100 amino acids. They are separated by an intron of 100 bp beginning after the codon for amino acid 10. Stimulin messenger RNA (mRNA) has 5' and 3' untranslated regions of 70 and 30 nucleotides, respectively. The poly A tail is 100 nucleotides in length. A complementary DNA (cDNA) made from mature stimulin RNA would have which of the following sizes? A) 500 bp B) 400 bp C) 300 bp D) 100 bp E) 70 bp

A) 500 bp

Membrane lipids have all of the following characteristics EXCEPT: A) Absence of charged groups B) Amphipathic C) Fatty acid esterified to glycerol D) Covalently bound oligosaccharides E) Extensive hydrophobic domains

A) Absence of charged groups

Microfilaments: What is the relevant cytoskeletal protein that forms or supports the structure. A) Actin B) Intermediate filament proteins C) Tubulin

A) Actin

Microvilli:What is the relevant cytoskeletal protein that forms or supports the structure. A) Actin B) Intermediate filament proteins C) Tubulin

A) Actin

Stress Fibers: What is the relevant cytoskeletal protein that forms or supports the structure. A) Actin B) Intermediate filament proteins C) Tubulin

A) Actin

What is a replication fork? A) An area where the DNA separates into two strands and DNA synthesis occurs. B) A molecular utensil required to stab nucleotides needed for DNA synthesis. C) The point where the cell must decide whether to replicate the DNA. D) A "factory" made of many enzymes that bind DNA for replication.

A) An area where the DNA separates into two strands and DNA synthesis occurs.

Eukaryotic mRNA molecules are processed and modified prior to transport out of the nucleus. Which of the follow modifications are involved in processing mRNA? A) Capping B) Poly T addition C) Splicing to remove exons D) All of the above E) None of the above

A) Capping

A 29-year-old patient visits her physician because she is having difficulty breathing. During the medical interview, the patient reports that she uses tobacco, usually smoking about ten cigarettes per day. The patient states that she is unsure about quitting smoking and experiences a lot of stress at work. According to the Transtheorectical Model of Change, which stage of change is it most appropriate for the physician to guide the patient? A) Contemplation B) Preparation C) Action D) Maintenance E) Pre-contemplation

A) Contemplation

If a completely radioactive double-stranded DNA molecule undergoes two rounds of replication in a solution free of radioactive label, what is the radioactivity status of the resulting four double-stranded DNA molecules? A) Half should contain no radioactivity B) All should contain radioactivity C) Half should contain radioactivity in both strands D) One should contain radioactivity in both strands E) None should contain radioactivity.

A) Half should contain no radioactivity

A four-year-old child who becomes easily tired and has trouble walking is diagnosed with Duchenne muscular dystrophy, an X-linked recessive disorder. Genetic analysis shows that the patient's gene for the muscle protein dystrophin contains a mutation in the promoter region. What would be the most likely effect of this mutation? A) Initiation of dystrophin transcription will be lacking. B) Termination of dystrophin transcription will be deficient. C) Capping of dystrophin mRNA will be defective. D) Splicing of dystrophin mRNA will be defective. E) Tailing of dystrophin mRNA will be defective.

A) Initiation of dystrophin transcription will be lacking.

The nucleolus is most obvious during which portion of the cell cycle? A) Interphase B) Mitosis C) Prophase D) Telophase E) Anaphase

A) Interphase

Consists of globular actin monomers linked into a double helix. A) Microfilament B) Intermediate filament C) Microtubule

A) Microfilament

Controls movement of proteins in and out of the nucleus: A) Nuclear pore complex B) Nucleolus C) Heterochromatin D) Outer nuclear membrane E) Euchromatin

A) Nuclear pore complex

An enzyme that utilizes NADH as a cofactor belongs to which class of enzymes? A) Oxidoreductase B) Transferase C) Hydrolase D) Lyase E) Isomerase F) Ligase

A) Oxidoreductase

Which of the following keeps the DNA polymerase on the DNA? A) PCNA B) General transcription factors C) MCM D) Pol δ E) RPA

A) PCNA

The chief function of a nucleolus is: A) Ribosomal RNA synthesis B) Storage of inactive DNA C) Transport of mRNA out of the nucleus D) Boundary between nucleus and cytoplasm E) Attachment site for spindle microtubules

A) Ribosomal RNA synthesis

What does the antiparallel nature of double-stranded DNA mean? A) The two strands run from 5' to 3' in opposite directions. B) The 5' ends of the two strands are on the same end of the double helix. C) The two DNA strands twist around each other in a helical pattern. D) The helix has a major and minor groove. E) The two strands of DNA are in anti-parallel universes.

A) The two strands run from 5' to 3' in opposite directions.

A 20-year-old anemic man is found to have an abnormal form of beta-globin (Hemoglobin Constant Spring) that is 172 amino acids long, rather than the 141 found in the normal protein. Which of the following point mutations is consistent with this abnormality A) UAA → CAA B) UAA → UAG C) CGA → UGA D) GAU → GAC E) GCA → GAA

A) UAA → CAA

A 17-year-old patient brought to the primary care clinic by her concerned mother. During the medical interview, the patient states that she really wants to lose more weight. The physician determines that the patient's body mass index (BMI) is 14.2. Within which weight status category does the patient's BMI fall? A) Underweight B) Normal C) Overweight D) Obese

A) Underweight

A 53-year-old man with a 48 pack-year history of cigarette smoking, develops a chronic cough. His physician order a chest x-ray and a "spot" is seen on his lung. He undergoes bronchoscopy and biopsy. He returns to the clinic to discuss the biopsy results. When the physician enters the exam room, the patient looks up and states "It is bad news, isn't it?" Which of the following is the most appropriate patient-centered response by the physician? A) Yes, it is. B) I'm so sorry. My father had lung cancer, too. C) It's important to stay hopeful and positive. D) You know, we can get you into a drug trial. E) I want you to see a grief counselor

A) Yes, it is.

Peptide bonds are what type of chemical bond? A. Amide B. Ester C. Ether D. Single E. Double

A. Amide

Joseph, a 6 year old boy, comes into your office complaining of pain in his right elbow. When taking the history the mother tells you the pain started right after Joseph tried running into the street after a baseball but thankfully his older brother was able to grab his wrist and pull him back onto the sidewalk. So you suspect Joseph suffered a "nursemaid injury". This injury involves pain to which ligament? A. Annular Ligament B. Radial Collateral Ligament C. Bicep Brachii Ligament D. Coracoacromial Ligament

A. Annular Ligament

A man got stabbed at the back of his arm in an anatomical space between the teres major, teres minor, and the long and lateral heads of the triceps brachii muscles. A nerve has been affected and now he is unable to abduct his arm to the horizontal. Identify what nerve is affected and what nerve roots? A. Axillary nerve, C5-6 B. Radial nerve, C5-T1 C. Suprascapular nerve, C5-6 D. Long thoracic nerve, C5,6,7

A. Axillary nerve, C5-6

A second semester Ross student is palpating for the brachial artery in order to take brachial artery pulse and is attempting to look for a tendon landmark to aid him. What is the tendon that the student is palpating for? A. Biceps Brachii tendon B. Flexor carpi radialis tendon C. Extensor Carpi radialis tendon D. Pronator teres tendon E. Extensor carpi radialis longus tendon

A. Biceps Brachii tendon

Which group of lymph nodes will ultimately receive the majority of lymph drainage from the breast? A. Central axillary lymph nodes B. Subdiaphragmatic lymph nodes C. Humeral lymph nodes D. Parasternal lymph nodes E. Subscapular lymph nodes

A. Central axillary lymph nodes

A 19-year-old man is brought to the hospital with a stab wound to the upper right portion of his chest. Upon further examination, you notice that the wound is superolateral to the pectoralis major and just medial to the deltoid. The bleeding appears to be a venous hemorrhage due to damage of which vessel? A. Cephalic Vein B. Axillary artery, 3rd division C. Axillary vein D. Common Carotid artery E. Jugular vein

A. Cephalic Vein

Immediately before draining into the axillary vein, the cephalic vein must pierce through this fascial layer: A. Clavipectoral Fascia B. Axillary Fascia C. Deep Cervical Fascia D. Costocoracoid membrane

A. Clavipectoral Fascia

A 45 year old woman comes to your office for her annual breast exam. She said she noticed her nipple looks strange with unusual folds. While doing your exam you notice her nipple is slightly retracted. Assuming she has a form of breast cancer, where is the cancer most likely to be found? A. Lactiferous ducts B. Suspensory ligaments C. Lactiferous sinuses D. Lymphatic ducts E. Retromammary space

A. Lactiferous ducts

What nerve pierces the pectoralis minor muscle? A. Medial Pectoral Nerve B. Lateral Pectoral Nerve C. Musculocutaneous Nerve D. Axillary Nerve

A. Medial Pectoral Nerve

A patient present to the clinic after falling off his skateboard while attempting to grind on a rail. He attempted to break his fall by grabbing the rail, but ended up taking the impact in his armpit. You perform a few tests and find out that the patient has paresthesia of the medial aspect of his forearm. Which nerve has been damaged and what are the spinal roots? A. Medial antebrachial cutaneous nerve - C8-T1 B. Median nerve - C5-T1 C. Medial brachial cutaneous nerve - C8-T1 D. Radial Nerve - C5-T1 E. Ulnar nerve - C7-T1

A. Medial antebrachial cutaneous nerve - C8-T1

A man presents to the emergency department after sustaining a fall from a ladder while hanging Christmas lights. He told the doctors that when he fell he tried to brace his fall and then heard a pop followed by pain. How would this patient most likely present after the doctor confirmed a fracture of the clavicle? A. Medial fragment of the clavicle is elevated B. Medial fragment is depressed C. Lateral fragment is elevated D. Shoulder is elevated due to unopposed Sternocleidomastoid muscle E. Shoulder is displaced laterally do to unopposed pectoralis major

A. Medial fragment of the clavicle is elevated

Which of the following proteins does not have a quaternary structure? A. Myoglobin B. Hemoglobin C. IgG D. Superoxide dismutase E. Collagen

A. Myoglobin

The nerve running through the triangular interval is a branch of what cord of the brachial plexus? A. Posterior B. Anterior C. Medial D. Lateral E. Superior

A. Posterior

A high school football player comes to the ER after sustaining a hard fall on his shoulder. Physical exam and X-ray show that the medial fragment of his clavicle is elevated, while the lateral fragment is depressed and pulled medially causing overriding. You suspect a clavicular fracture. The pull of what muscle causes the medial fragment to be elevated? A. Sternocleidomastoid B. Pectoralis Minor C. Triceps D. Trapezius E. Rhomboid major F. Pectoralis major

A. Sternocleidomastoid

A woman visits her doctor with the complaint of being unable to lift her arm off her body. She says that once her arm is elevated slightly, then she can raise her arm the rest of the way, but the initial movement is where she's having trouble. What nerve is likely impaired to prevent her from using this muscle? A. Suprascapular Nerve B. Upper Subscapular Nerve C. Axillary Nerve D. Medial Pectoral Nerve E. Lateral Pectoral Nerve

A. Suprascapular Nerve

A college baseball pitcher is presenting to his team's physician with trouble rotating his arm while warming up during practice. Which shoulder muscle is LEAST likely to play a role in his presentation. A. Supraspinatus muscle B. Infraspinatus muscle C. Subscapularis muscle D. Teres Minor muscle

A. Supraspinatus muscle

Which of the following arteries does not participate in the arterial anastomosis around the scapula? A. Transverse cervical B. Dorsal Scapular C. Subscapular D. Circumflex scapular

A. Transverse cervical

Jealous of your neighbor's Christmas lights, you decide to go up on the roof and put as many lights as you can. Not paying attention, you slid on a piece of ice and fall off the roof and land on the ground injuring your shoulder. The ER physician described your clinical presentation as "Waiter's Tip" Deformity. Describe which brachial plexus trunk and injured and its clinical presentation? A. Upper Trunk (C5-C6); upper limb is adducted, medially rotated arm and extended elbow B. Upper Trunk (C5-C6); upper limb is abducted, medially rotated arm and extended elbow C. Upper Trunk (C5-C7); upper limb is adducted, medially rotated arm and extended elbow D. Upper Trunk (C5-C7); upper limb is abducted, medially rotated arm and extended elbow

A. Upper Trunk (C5-C6); upper limb is adducted, medially rotated arm and extended elbow

A 25 year old woman has just given birth to her first son, her labour was very difficult and she seems to have concern about the position of her newborn son's right arm. As the years progressed, she noticed that her son is unable to lift his right arm. He was diagnosed with Erb's palsy and she is considering suing the physician for the excessive shoulder pulling of her infant during labor. What position did she find her son's arm in and what nerve roots were damaged? A. adducted and medially rotated, C5-C6 B. abducted and laterally rotated, C5-C6 C. adducted and medially rotated, C5,C6,C7,C8 D. abducted and laterally rotated, C5-C6 E. adducted and laterally rotated, C5, C6, C7

A. adducted and medially rotated, C5-C6

A building block of microfilaments:

Actin

Lesion to the Dorsal Scapular nerve would lead to: A. Complete loss of scapular movement B. Hypertrophy of the supraspinatous muscle C. Inability to superiorly rotate the scapula D. Diminished movement of the scapula

D. Diminished movement of the scapula

A man is brought into the Emergency Room after being thrown from a motorcycle. Upon examination, you notice that his left arm hangs by his side with medial rotation. You know this to be an injury to the superior trunk of the brachial plexus. The name of this presentation is called: A. Claw hand B. Horner's syndrome C. Terrible triad D. Erb's palsy E. Bell's palsy

D. Erb's palsy

A patient comes to your office with complaints of paresthesia of the medial part of the palm and medial one and a half fingers. She tells you that it all started when she fell on her elbow. From her chief complaint and history, and even before doing an x-ray, you already know that is likely to have: A. Fractured her medial epicondyle and injured her median nerve B. Fractured her lateral epicondyle and injured her ulnar nerve C. Fractured her lateral epicondyle and injured her radial nerve D. Fractured her medial epicondyle and injured her ulnar nerve E. Fractured her humerus at the supracondylar line and injured her median nerve

D. Fractured her medial epicondyle and injured her ulnar nerve

Injury to the Radial nerve most likely correlates with which of the following Humeral fractures? A. Anatomical Neck Fracture B. Surgical Neck Fracture C. Distal Humerus Fracture D. Humeral Shaft Fracture E. Nursemaid's Fracture

D. Humeral Shaft Fracture

The breast receives blood supply directly from what arterial branches? A. Medial thoracic and long thoracic arteries B. Axillary, suprascapular and long thoracic arteries C. Thoracic aorta, subclavian and dorsal scapular arteries D. Internal thoracic, lateral thoracic, posterior intercostal arteries

D. Internal thoracic, lateral thoracic, posterior intercostal arteries

Mr. Johnson, a 35 year old Caucasian race car driver, arrives in the emergency room after a car crash. Initial examination reveals hyperextension of the neck and a numb, tingling feeling throughout his left leg. After radiological analysis, the vertebral disk between L3/L4 is found to be ruptured. What nerve do you suspect is compressed and what spinal ligament do you suspect is damaged? A. L3, anterior longitudinal ligament B. L3, posterior longitudinal ligament C. L3, supraspinous ligament D. L4, anterior longitudinal ligament E. L4, posterior longitudinal ligament

D. L4, anterior longitudinal ligament

A 45-year-old white male presents in your office complaining of lower back pain that started while lifting a couch at work a few days prior. After performing a physical exam, you suspect a herniated disc, but to be sure you order an MRI. The MRI confirms the man has a herniated L4/L5 intervertebral disc. What nerve is most likely compressed causing your patients pain? A. L2 B. L3 C. L4 D. L5

D. L5

A common deficiency in what digestive enzyme can lead to bloating, diarrhea, flatulence and nausea after drinking milk? A. Sucrase B. Maltase C. Isomaltase D. Lactase E. Amylase

D. Lactase

An elderly man at a nursing home is known to have a degenerative brain disease. When cerebrospinal fluid (CSF) is withdrawn by lumbar puncture for further examination, which of the following structures is most likely penetrated by the needle during the procedure? A. Pia mater B. Filum terminale externum C. Posterior longitudinal ligament D. Ligamentum flavum E. Annulus fibrosus

D. Ligamentum flavum

Much of the functional machinery of the cell is carried out by macromolecular complexes. Most of these complexes are a result of the polymerization of monomeric groups into large, covalently linked molecules. Which type of complex is an exception by being a non-covalent assembly? A. Glycogen and starch B. Proteins C. DNA D. Lipid membranes

D. Lipid membranes

A 22 year-old man presents with a stab wound to the lateral chest beneath his left arm. Physical exam reveals that the medial border of the patient's left scapula projects posteriorly. Which nerve is most likely damaged? A. Axillary nerve B. Medial pectoral nerve C. Radial nerve D. Long thoracic nerve E. Suprascapular nerve

D. Long thoracic nerve

After having a mastectomy for the removal of an extensive breast cancer, the 34yr old patient tells her physician that she has great difficulty abducting her arm above the level of her shoulder. Physical examination reveals that she has a "winged" scapula when she pushes against the wall. What structure has been injured during her breast surgery to result in this problem? A. Axillary nerve B. Pectoralis major muscle C. Deltoid muscle D. Long thoracic nerve E. Lateral pectoral nerve

D. Long thoracic nerve

A 38-year-old male sustains a "crush" injury to the inferior angle of his scapula. He is experiencing difficulty medially rotating his arm. What nerve do you expect has been damaged in this patient? A. Dorsal Scapular B. Suprascapular C. Spinal Accessory D. Lower Subscapular E. Axillary

D. Lower Subscapular

While doing an axillary lymph node dissection, a surgeon cleans the space between the pectoralis major and minor muscles in an attempt to remove all of the pectoral lymph nodes. Afterwards, it is noted that the patient's lower portion of the pectoralis major is paralyzed. Which nerve was most likely injured during the surgery? A. Dorsal scapular nerve B. Axillary nerve C. Lateral Pectoral nerve D. Medial Pectoral nerve E. Suprascapular nerve

D. Medial Pectoral nerve

Upon neurological testing of a patient, the patient reveals no cutaneous sensation over the lateral aspect of forearm. Such a finding would indicate injury to some portion of which nerve? A. Median nerve B. Axillary nerve C. Inferior trunk D. Musculocutaneous nerve E. Inferior trunk F. Ulnar

D. Musculocutaneous nerve

A 76- year- old man from Arkansas, a retired plumber, presents to his health care provider complaining of pain in his left shoulder for 4 days that has been getting progressively worse. He is diabetic and hypertensive and also has chronic osteoarthritis. He suffers from hay fever and had an appendectomy for a ruptured appendix at age 25, and replacement of his left hip at age 70 because of severe osteoarthritis. Systematic enquiry reveals that he also has occasional constipation. He is currently on antihypertensive medication and low dose aspirin. On examination he is a thin white male in no apparent distress. His blood pressure is elevated but examination of his heart and lungs is normal. Laboratory investigation reveals elevated blood glucose. Which of the following best describes his chief complaint? A. Diabetes B. Hypertension C. Osteoarthritis D. Pain in the left shoulder E. Constipation

D. Pain in the left shoulder

Patient came to your clinic with a lump on the lateral aspect of her breast. After X-rays and CT scan, you determined that the lump was likely cancerous. If the cancer spreads via the lymphatic system, which of the following illustrates the most likely pathway for the cancer cell metastasis? A. Pectoral - Apical - Central - Humeral B. Humeral - Central - Apical - Supraclavicular C. Pectoral - Central - Subscapular - Suprascapular D. Pectoral - Central - Apical - Supraclavicular E. Parasternal - Subdiaphragmatic - Thoracic duct

D. Pectoral - Central - Apical - Supraclavicular

A woman presents to your clinic with a breast complaint. She states that she has noticed one breast being lower than the other for the past few months. Upon examination you notice that when her hands are placed on hips and pressed forward, the left breast elevates. You are concerned that she has breast cancer located to which of the following structure? A. Lymphatic drainage B. Suspensory ligaments C. Lactiferous ducts D. Pectoral fascia

D. Pectoral fascia

One of four major phosphoglycerolipids acts a precursor in calcium signaling. Name it. A. Phosphatidyl Choline B. Phosphatidyl Ethanolamine C. Phosphatidyl Serine D. Phosphatidyl Inositol E. Triacylglyceride

D. Phosphatidyl Inositol

Which amino acid is considered a helical breaker? A. Alanine B. Glycine C. Phenylalanine D. Proline E. Methionine

D. Proline

During a surgery in the axillary region, you accidentally sever the patient's musculocutaneous nerve. When they awake you notice that they can still perform some flexion at the elbow. What innervates the muscle that is performing this flexion? A. Ulnar nerve B. Anterior interossesus nerve C. Axillary nerve D. Radial nerve E. Ventral rami of C4

D. Radial nerve

In the above vignette, the patient begins to tear up, saying, "I worry so much about being able to provide for my child". The doctor responds by saying, "you really want to be a good parent, and having this job is very important to you". He is practicing active listening, and the facilitating technique used is best described as: A. Paraphrasing B. Parroting C. Repeating D. Reflecting E. Mirroring

D. Reflecting

A 50 year old man was just diagnosed with a degenerative nerve disease and notices a change in the position of his scapula. He goes to his physician and is noted to have a change in the position of his scapula; it is moved laterally with a raised medial border. His physician thinks there is damage to the dorsal scapular nerve. What is the patient likely to be suffering from? A. Trapezius Palsy B. Triceps Palsy C. Erb's Palsy D. Rhomboid Palsy

D. Rhomboid Palsy

The long thoracic nerve innervates which muscle: A. suprascapulis B. Middle scalene C. Anterior scalene D. Serratus anterior E. Teres major

D. Serratus anterior

What 3 muscles attach to the Coracoid Process? A. Short head of biceps brachii, pec major, coracobrachialis B. Long head of biceps brachii, pec minor, coracobrachialis C. Long head of biceps brachii, short head of triceps, coracobrachialis D. Short head of biceps brachii, pec minor, coracobrachialis E. Short head of biceps brachii, lateral head of triceps, coracobrachialis

D. Short head of biceps brachii, pec minor, coracobrachialis

A 25-year-old college football player injured his shoulder while diving to catch a football in mid air. As he came down he slammed his shoulder into the ground with all his weight. He was in tremendous pain so he went to the ER. An X-ray showed that the distal end of the clavicle had been displaced superiorly and the shoulder had dropped inferiorly. What is this presentation called and what ligament(s) is/are the most likely affected? A. Drooped shoulder, coracoclavicular B. Dislocation of coracoclavicular, coracoclavicular C. Drooped shoulder, acromioclavicular D. Shoulder separation, acromioclavicular & coracoclavicular

D. Shoulder separation, acromioclavicular & coracoclavicular

A twenty year old man is in the emergency room suffering a knife wound. He complains of weakness upon elevation of his shoulder. The main muscle responsible for this action is innervated by? A. Dorsal scapular nerve B. Dorsal primary rami C. Transverse cervical nerve D. Spinal accessory nerve

D. Spinal accessory nerve

A landscaper presents to his primary care physician describing extreme difficulty when starting his pull start lawnmower and turning his arm towards his body. He says it began as just weakness that has now progressed to an inability to start his lawn mower, and he decided to come in when his wife noticed his right forearm muscles beginning to "shrink." When the physician measures the left forearm he finds that its circumference is significantly larger, and that the patient has loss of sensation to the back of his arm extending over his elbow to his thumb, and first two fingers but NOT the tips. His physician sends him to a medical imaging center for a chest x-ray and notes a mass compressing his axillary sheath and makes a diagnosis of a progressive Pancoast Tumor. What Portion of the Brachial plexus is most likely affected? A. Anterior Cord B. Medial Cord C. Posterior Cord D. Radial Nerve of the Posterior Cord E. Ulnar Nerve of the Medial Cord F. Supraspinous nerve of the Anterior Cord

C. Posterior Cord

The intrinsic muscles of the back move the vertebral column and maintain posture. What nerves are these deep muscles innervated by? A. Anterior Rami B. Thoracodorsal Nerve C. Posterior Rami D. Ventral Rami

C. Posterior Rami

A 26-year-old male comes to your office with the inability to extend his wrist. What nerve is most likely damaged giving this clinical presentation? A. Ulnar nerve B. Musculocutaneous nerve C. Radial nerve D. Median nerve E. Axillary nerve

C. Radial nerve

Mr. Smith comes into the emergency department after a traumatic fall and consequently fracturing the mid-shaft of his humerus. Mr. Smith tells you that he cannot extend his forearm. What is the nerve responsible for the extension of the forearm? A. Axillary nerve B. Musculocutaneous C. Radial nerve D. Ulnar nerve

C. Radial nerve

A patient presents to the ER holding his right arm. He tells you that he was riding his bike when he hit a curb and went over the handle bars and landed on his arm. You order an x-ray and it comes back with mid shaft humeral fracture. What nerve are you most concerned about and what is an associated clinical sign? A. Median nerve - hand of benediction B. Ulnar nerve - claw hand C. Radial nerve - wrist drop D. Median nerve - ape hand E. Radial nerve - inability to extend forearm against resistance

C. Radial nerve - wrist drop

A pediatric patient is brought into your ER with pain in the elbow region. You assess the patient and determine a subluxation and dislocation of the radius. What is causing the pain? A. Tearing of the annular ligament B. Avulsion fracture of the radius C. Radius compressing the annular ligament D. Torn radial nerve

C. Radius compressing the annular ligament

Which muscle forms the superomedial border of the suboccipital triangle? A. Obliquus Capitis Superior B. Semispinalis Capitis C. Rectus Capitis Posterior Major D. Obliquus Capitis Inferior E. Rectus Capitis Posterior Minor

C. Rectus Capitis Posterior Major

A 24-year-old male presents to the emergency room after being stabbed in the back of the head with a broken beer bottle during a bar fight. His suboccipital nerve appears to be damaged. What muscles comprise the borders of the triangle in which the suboccipital nerve is found? A. Rectus capitis minor, Obliquus capitis superior, Obliquus capitis inferior B. Rectus capitis major, Obliquus capitis superior, Semispinalis capitis C. Rectus capitis major, Obliquus capitis superior, Obliquus capitis inferior D. Rectus capitis minor, Obliquus capitis inferior, Semispinalis capitis

C. Rectus capitis major, Obliquus capitis superior, Obliquus capitis inferior

A female patient has abdominal pain and visits the emergency department. She has a history of past sexual abuse and when she learns that she must have a pelvic exam, she requests a female physician. The male physician performing her evaluation says, "I'm sorry, that's not going to happen. I'm just too busy to go tracking down someone for that." The patient refuses to be examined and leaves. A diagnosis of ruptured ectopic pregnancy is missed. Which of the following ACGME competencies is most lacking in this scenario? A. Systems-Based Practice B. Professionalism C. Relationship-Centered Care D. Improvement in Practice F. Personal and Professional Development

C. Relationship-Centered Care

A 65 year old woman is diagnosed with breast cancer. When asked to place her hands on her hips and push her arms forward, her breast moves up upon contraction of the pectoralis muscles. What space has the cancer invaded? A. Axillary space B. Parasternal space C. Retromammary space D. Quadrangular space

C. Retromammary space

What is the landmark where the subclavian artery changes name to the axillary artery? A. Rib 2 B. Teres major C. Rib 1 D. Teres minor

C. Rib 1

A young man fell down a flight of stairs, avulsing the lesser tubercle of the humerus. Which of the following structures would most likely have structural and functional damage? A. Supraspinatus B. Infraspinatus C. Subscapularis D. Teres minor E. Latissimus dorsi

C. Subscapularis

A patient presents to the emergency room with a gunshot wound immediately inferior to the right clavicle. A CT scan shows that the bullet did not damage the axillary artery itself, but it did lacerate a branch of this artery. This branch typically arises immediately lateral to the first rib, well medial to the pectoralis minor muscle. Which artery is most likely damaged in this patient? A. Subscapular artery B. Suprascaular artery C. Superior thoracic artery D. Thoracoacromial trunk E. Lateral thoracic artery

C. Superior thoracic artery

A woman comes into a blood bank to donate some of her very sought after O negative blood. The nurse inserts the needle into her medial cubital vein. As the nurse inserts the needle, she uses landmark borders to get to the right structure. Which of the following is NOT a border of the superficial cubital fossa? A. Imaginary line between lateral and medial epicondyles of the humerus B. Brachioradialis muscle C. Tendon of the biceps brachii D. Pronator teres muscle

C. Tendon of the biceps brachii

Disulfide bonds stabilize principally this type of structure in proteins. A. Primary B. Secondary C. Tertiary D. Quaternary E. Macromolecular Assembly

C. Tertiary

A patient is rushed to the emergency room after getting in a street fight. The patient was stabbed with a knife, which inflicted a deep penetrating wound. If the wound extended deep through the distal end of pectoralis minor, which of the following arteries is most as risk of being severed? A. Thoracoacromial artery B. Superior thoracic artery C. Thoracodorsal artery D. Posterior circumflex humeral artery

C. Thoracodorsal artery

Which base is found in DNA and not RNA? A. Adenine B. Cytosine C. Thymine D. Uracil E. Guanine

C. Thymine

In the above vignette, after discussing her primary concerns, the doctor states, "Ok. Are your parents dead?" This is a poor example of what kind of statement? A. Open ended B. Facilitating C. Transitional D. Summarizing E. Rapport-building

C. Transitional

The process of converting the information stored in DNA to a protein-encoded functional expression of that information goes through a mRNA intermediate. What is the process of generating the protein from the mRNA called? A. Replication B. Transcription C. Translation D. Modification E. Secretion

C. Translation

Paul was playing a game of baseball and in the process, fell with his arms extended and abducted. His father being a medical doctor was worried that he suffered a posterior dislocation of the elbow joint. What nerve is likely to be damaged in this injury? A. Radial nerve B. Musculocutaneous nerve C. Ulnar nerve D. Axillary nerve

C. Ulnar nerve

If a person is stabbed in the quadrangular space, what two structures are most likely to be injured?

Axillary nerve and posterior circumflex humeral artery

Analysis of a cell line that rapidly transforms into a tumor cell line demonstrated an increased mutation rate within the cell. Further analysis indicated that there was a mutation in the DNA polymerase enzyme that synthesizes the leading strand. This inactivating mutation is likely to be in which of the following activities of this DNA polymerase? A) 5'- exonuclease activity B) 3'- exonuclease activity C) Phosphodiester bond making capability D) Uracil-DNA glycosylase activity E) Ligase activity

B) 3'- exonuclease activity

The following mutation occurred near the 5' end of the coding sequence. The sequence shown is the coding strand of the gene. It includes the start codon and the mutation. G A T T C A T G T T C G G A C T G G T G T A T G A T T C A T G T T C G G A C T G T A T What is the consequence of this mutation? A) The amino end of the encoded protein is normal, but the sequence between the site of the mutation and the C terminus is completely abnormal B) A single amino acid is missing in the sequence C) A single amino acid is replaced by another amino acid D) This is a nonsense mutation that results in premature chain termination E) Splicing of the transcript is abnormal

B) A single amino acid is missing in the sequence

Which of the following codons is the start codon? A) AUU B) AUG C) CCC D) UAA E) UAG

B) AUG

Variations in size of the nucleolus are primarily due to variation in the A) Rate of DNA synthesis B) Amount of granular component C) Amount of fibrous component D) Amount of histone turnover E) Rate of cell motility

B) Amount of granular component

A patient displays tiredness and lethargy, and blood work demonstrates an anemia. Western blot analysis indicates significantly greater levels of beta-globin than beta-globin. Molecular analysis indicates a single nucleotide change in an intron of the beta-globin gene. How does such a mutation lead to this clinical finding? A) A microRNA is produced, which is target against the beta-globin mRNA, thereby reducing beta- globin production. B) Creation of an alternative splice site, such that beta-globin levels are decreased. C) Creation of a new transcription initiation site, such that the mRNA for betea-globin is now out of frame. D) Creation of a stop codon in the beta-globin mRNA. E) Elimination of the polyadenylation signal, thereby reducing beta-globin production.

B) Creation of an alternative splice site, such that beta-globin levels are decreased.

Cholesterol functions in the plasmalemma to: A) Increase fluidity of the lipid bilayer. B) Decreases fluidity of the lipid bilayer C) Facilitate the diffusion of ions through the lipid bilayer D) Assists in the transport of hormones across the lipid bilayer E) Bind extracellular matrix molecules

B) Decreases fluidity of the lipid bilayer

Under conditions of active exercise, protein synthesis is reduced in the muscle. Under these conditions, which aspect of translation is inhibited? A) Initiation of translation B) Elongate during translation C) Termination of translation D) Synthesis of mRNA E) Production of rRNA

B) Elongate during translation

What would NOT be found either in, or bound to, the leaflet of the plasma membrane facing the cytoplasm? A) Phosphatidylserine B) Glycolipid C) Cholesterol D) Peripheral membrane protein E) Integral membrane protein

B) Glycolipid

Provide structural supports to astrocytes. A) Microfilament B) Intermediate filament C) Microtubule

B) Intermediate filament

Fingernails: What is the relevant cytoskeletal protein that forms or supports the structure. A) Actin B) Intermediate filament proteins C) Tubulin

B) Intermediate filament proteins

All of the following statements concerning membrane composition are true EXCEPT: A) Plasma membranes have more lipids than do mitochondrial inner membranes B) Intracellular membranes have less protein than does myelin C) Cholesterol is present in plasma membranes and intracellular membranes. D) Cholesterol composition affects membrane fluidity E) Cell membranes have no free carbohydrate moieties

B) Intracellular membranes have less protein than does myelin

Which of the following is true about the lagging strand? A) It is synthesized as one continuous strand, making it lag behind the other strand, which is made in many pieces at once. B) It is synthesized as a series of Okazaki fragments. C) It is more heavily coated with SSB proteins, slowing down synthesis. D) It is synthesized more slowly due to the presence of the sliding clamp.

B) It is synthesized as a series of Okazaki fragments.

Which of the following membranes has the highest ratio of lipid-to-protein? A) Red cell membrane B) Myelin C) Golgi membranes D) Rough endoplasmic reticulum E) Outer mitochondrial membrane

B) Myelin

A 3-year-old boy, whose parents did not immunize him due to fears of postimmunization side effects, exhibited fever, chills, severe sore throat, lethargy, trouble breathing, and a husky voice. Physical exam indicated greatly enlarged lymph nodes, an increased heart rate, and swelling of the palate. A picture of the boy's throat is shown below. The throat is dull red, and a gray exudate (pseudomembrane) is present on the uvula, pharynex, and tongue. A necessary cofactor for these symptoms to appear in this child is which of the following? A) ATP B) NAD+ C) FAD D) Acetyl-CoA E) UDP-glucose

B) NAD+

DNA contains which one of the following components? A) Nitrogenous bases joined by phosphodiester bonds B) Negatively charged phosphate groups on the exterior of the molecule C) Base pairs stacked along the exterior of the axis of the molecule D) Two strands that run in the same direction E) The sugar ribose

B) Negatively charged phosphate groups on the exterior of the molecule

An inclusion not bound by a membrane that is observable only during interphase: A) Nuclear pore complex B) Nucleolus C) Heterochromatin D) Outer nuclear membrane E) Euchromatin

B) Nucleolus

Which of the following post-translational modifications is most frequently used to rapidly alter protein function? A) Acetylation B) Phosphorylation C) Carboxylation D) Glycosylation E) Farnesylation

B) Phosphorylation

A 19-year-old patient presents to the primary care clinic with the chief complaint of having the flu. During routine questioning, the patient discloses that he is getting ready for a ski trip to Colorado. The physician advises the patient to wear a ski helmet or protective headgear. Which of the following best describes the level of prevention activity exercised by this physician? A) Primordial B) Primary C) Secondary D) Tertiary E) Quaternary

B) Primary

DNA is duplicated in the cell cycle during: A) G2 phase B) S phase C) M phase D) G1 phase E) G0 phase

B) S phase

In the read-out of the genetic code in eukaryotes, which one of the following processes acts before any of the others? A) tRNAi alignment with mRNA B) Termination of transcription C) Movement of the ribosome from one codon to the next D) Recruitment of termination factors to the A site E) Export of mRNA from the nucleus

B) Termination of transcription

Splicing best refers to which of the following statements? A) The removal of promoter sequences from genes B) The removal of introns from transcripts C) The removal of origins of replication (ori) D) The removal of exons from transcripts E) Product from an info-mercial (it splices, it dices..)

B) The removal of introns from transcripts

Poliovirus encodes a protease that hydrolyzes a specific eukaryotic translation factor. The consequences of this digestion result in inhibition of translation from eukaryotic capped mRNAs but not the translation of viral encoded RNAs. How do the viral RNAs manage to be translated? A) The cap of the viral encoded RNAs does not need any initiation factors to be recognized and enter the small ribosomal subunit onto the RNA and find the start codon. B) The viral encoded RNAs recognized the initiation factor that is cleaved and uses that to enter the small ribosomal subunit onto the RNA and find the start codon. C) The virus makes so many viral mRNA that it titers out the initiation factors of the cell. D) The virus knocks out the cellular ribosome and replaces it with a virally encoded ribosome. E) The viral encoded mRNAs are simpler to bind to by the small ribosomal subunit than the cellular mRNAs.

B) The viral encoded RNAs recognized the initiation factor that is cleaved and uses that to enter the small ribosomal subunit onto the RNA and find the start codon.

A geneticist introduces the genes for a super-spliceosome into the human germ line that is able to remove every intron without fail. In which way would people with this genetic innovation differ from the rest of the species? A) They would have only one kind of histone. B) They would produce fewer unique proteins. C) They would transcribe fewer genes. D) They would produce mRNA transcripts without cap and poly-A tail. E) Their proteins would no longer be modified post-translationally.

B) They would produce fewer unique proteins.

The 5' cap and the 3' poly-A tail are added in the production of mature eukaryotic mRNA. The 5' cap and the 3' poly-A tail have which respective functions? A) Transcript stability and transcriptional termination B) Translation initiation and transcript stability C) Transcriptional initiation and transcriptional termination D) Translation initiation and translation termination E) The functions have not been determined

B) Translation initiation and transcript stability

One of the ways that cells communicate with each other is by secretion of various molecules. The secreted molecule is known as: A) a receptor molecule B) a signal molecule C) a spectrin tetramer D) an integrin E) an anticodon

B) a signal molecule

An organic substance bound to an enzyme and essential for its activity is called A) isoenzyme B) cofactor C) holoenzyme D) apoenzyme.

B) cofactor

Which one of the following statements regarding enzyme inhibition is correct? A) competitive inhibition is seen when a substrate competes with an enzyme for binding to an inhibitor protein B) competitive inhibition is seen when the substrate and the inhibitor compete Tor the active site on the enzyme C) non-competitive inhibition of an enzyme can be overcome by adding large amount of substrate D) non-competitive inhibitors often bind to the enzyme irreversibly.

B) competitive inhibition is seen when the substrate and the inhibitor compete Tor the active site on the enzyme

The newly formed erythrocyte does not have any DNA. However, it can still control gene expression. One of the ways of controlling the synthesis of hemoglobin proteins is by the availability of heme. When heme is low a kinase called heme controlled inhibitor (HCI) is active. What translation initiation factor is phosphorylated by HCI thereby making it difficult to translate the globin mRNA in the cell? A) eIF1 B) eIF2 C) eIF2B D) eIF3 E) eIF4E

B) eIF2

What is a substance that would probably never pass through nuclear pores in an interphase nucleus? A) mRNA B) tubulin C) histones D) tRNA E) ribosomal protein

B) tubulin

In the above vignette, after the physician's question, the mother yells, "You were a half-hour late and I have a screaming baby with me-that is just unprofessional!" An empathic reply would be: A. "Sorry, we just overbooked" B. "It must be very frustrating to have to wait while your child is upset" C. "That's true, but you really shouldn't bring a child to an adult physical exam" D. "Can I bring you some coffee?" E. "Is there someone who can watch your child for you while we visit?"

B. "It must be very frustrating to have to wait while your child is upset"

Patient comes into the emergency room and is bleeding profusely from his shoulder. He claims he was at a bar watching a football game when all hell broke loose. He was stabbed several times near his right shoulder in his axillary region. The physician determines that the stab wound damaged pectoralis minor. He determined this by noticing instability in movement of the scapula. What artery is most likely damaged? A. 1st part of the axillary artery B. 2nd part of the axillary artery C. 3rd part of the axillary artery D. 4th part of the axillary artery

B. 2nd part of the axillary artery

A weightlifter is making loud grunting noises in the gym as he performs bicep curls with 100-lb dumbbells in each hand. Suddenly, a loud "pop" is heard all over the gym, and he drops the weights and screams in agony. As an aspiring doctor, you run over and seize the opportunity to perform a general survey. What might you notice? A. Posterior dislocation of the elbow joint B. A bulge in the center of the anterior arm C. Edema over the pectoral region D. "Dinner fork" deformity E. A possible "Smith" fracture

B. A bulge in the center of the anterior arm

A patient presents to your office unable to make a complete fist with his right hand. He said that he was climbing a tree to save a kitten when a branch broke from under him. He tried to hang on for a couple minutes but had to let go. When he tried grabbing the kitten that was about to run away, he noticed his medial two fingers would not bend. What spinal nerve roots were damaged during this accident? A. C5, C6, C7 B. C8, T1 C. C6, C7, C8 D. C1, C2, C3

B. C8, T1

What vein pierces the clavipectoral fascia? A. Axillary vein B. Cephalic vein C. Subclavian vein D. Brachial vein E. Basilic vein

B. Cephalic vein

A man with hypertension has a blocked axillary artery. When a surgeon decided to do an angioplasty, he asked a 1st semester student why there was still blood supply to the scapula. Which two arteries allow for blood to be supplied to the scapula? A. Subscapular and Circumflex scapular B. Dorsal scapular and Suprascapular C. Deltoid and Acromial D. Brachial and Intercostal E. Subclavian and Tranverse Cervical

B. Dorsal scapular and Suprascapular

After a complicated delivery, the mother notices that her baby has a waiter's tip deformity. With her nursing background, she knows it means an upper trunk brachial plexus injury. What is the other name for this condition, and what nerve roots are damaged? A. Shoulder dystocia, C8-T1 B. Erb-Duchenne Palsy, C5-C6 C. Klumpke paralysis, C5-C6 D. Prefixed brachial plexus, C4-C8

B. Erb-Duchenne Palsy, C5-C6

A 16-year-old boy is brought to the doctor complaining of fever, chills, and a stiff neck. The doctor suspects a case of meningitis and requests a lumbar puncture to be performed. Which layers does the needle pass through? A. Skin, posterior longitudinal ligament, anterior longitudinal ligament, dura mater, arachnoid mater B. Skin, ligamentum flavum, epidural space, dura mater, arachnoid mater C. Skin, ligamentum flavum, dura mater, epidural space, arachnoid mater D. Skin, posterior longitudinal ligament, ligamentum flavum, dura mater, arachnoid mater, pia mater

B. Skin, ligamentum flavum, epidural space, dura mater, arachnoid mater

The dorsal scapular artery is part of the anastomosis around the scapula, which artery does it typically, directly branch from? A. Thyrocervical trunk B. Subclavian artery C. Subscapular artery D. Axillary artery

B. Subclavian artery

The suboccipital triangle contains the vertebral artery and which nerve that has ONLY motor innervation fibers? A. Greater occipital nerve B. Suboccipital nerve C. Least occipital nerve D. Thoracodorsal nerve E. Spinal accessory nerve

B. Suboccipital nerve

Which of the following branches of the Axillary artery contribute to anastomoses around the Scapula? A. Suprascapular artery B. Subscapular artery C. Dorsal scapular artery D. Posterior circumflex humeral artery E. Lateral thoracic artery

B. Subscapular artery

Which one of the following muscles make up the "floor" of the cubital fossa? A. Biceps brachii tendon B. Supinator muscle C. Brachioradialis D. Pronator teres

B. Supinator muscle

A patient comes to your office complaining of pain in his right shoulder after a skateboarding accident. Upon examination, you ask your patient to hold his right upper limb out at 90 degrees, and observe that in order to do so, he must swing his arm. What nerve is most likely damaged seeing this presentation? A. Axillary Nerve B. Suprascapular Nerve C. Infraspinatus Nerve D. Long Thoracic Nerve

B. Suprascapular Nerve

With an injury that causes the 'waiter's tip deformity (Erb's Palsy), which nerve deficit causes the humerus to medially rotate? A. Musculocutaneous Nerve B. Suprascapular Nerve C. Dorsal Scapular Nerve D. Ulnar Nerve E. Lateral Pectoral Nerve

B. Suprascapular Nerve

The rotator cuff is a group of muscles which surround the shoulder joint to stabilize it in addition to their actions on the upper limb. The nerves that supply these muscles are: A. Axillary, radial, musculocutaneous, long thoracic nerves B. Suprascapular, axillary, upper subscapular, lower subscapular nerves C. Upper subscapular, lower scapular, medial pectoral and lateral pectoral nerves D. Suprascapular, axillary, medial pectoral and lateral pectoral nerves E. Musculocutaneous, long thoracic, suprascapular, axillary nerves

B. Suprascapular, axillary, upper subscapular, lower subscapular nerves

Which of the following muscles is NOT considered a part of the rotator cuff muscles? A. Supraspinatus B. Teres major C. Infraspinatus D. Subscapularis E. Teres minor

B. Teres major

A lesion of the lateral cord will lead to severe weakness in: A. Pectoralis minor B. The upper arm flexors C. The upper arm extensors D. The deltoid and teres minor E. Supraspinatus and infraspinatus

B. The upper arm flexors

The breast receives blood supply directly from arteries that are branches from which larger arteries? A. Medial and lateral thoracic arteries B. Thoracic Aorta, Subclavian and Axillary artery C. Thoracic Aorta and axillary artery D. Axillary, subclavian and long thoracic artery

B. Thoracic Aorta, Subclavian and Axillary artery

The acromial artery is a direct branch of which artery? A. Subscapular artery B. Thoracoacromial artery C. Pectoral artery D. Superior thoracic artery E. Deltoid artery

B. Thoracoacromial artery

What is the correct pathway for the vertebral artery? A. Travels through intervertebral foramen of C1-C3, passes laterally along vertebral column, passes superiorly through foramen magnum B. Travels through transverse foramen of C1-C6, passes medially along the posterior arch of C1, passes superiorly through foramen magnum C. Travels through transverse foramen of C1-C3, passes medially along the posterior arch of C1, passes superiorly through foramen magnum D. Travels through transverse foramen of C1-C6, passes laterally along the vertebral column, passes superiorly through foramen magnum

B. Travels through transverse foramen of C1-C6, passes medially along the posterior arch of C1, passes superiorly through foramen magnum

A patient presents to the clinic with a stab wound to the right upper arm. Upon examination it is noted that there is pulsatile bleeding coming from the wound. The wound itself is found to be between the teres minor, teres major, and medial to the long head of the triceps. What is this space called and what blood vessel was hit? A. Triangular space - dorsal scapular artery B. Triangular space - circumflex scapular artery C. Triangular interval - profunda brachii D. Triangular space - profunda brachii E. Triangular interval - circumflex scapular

B. Triangular space - circumflex scapular artery

Oxidative phosphorylation utilizes oxygen to form what molecule? A. CO2 B. Water C. NAD+ D. Protons E. CoQ

B. Water

A 46-year-old female came to her OBGYN and was diagnosed with breast cancer with the tumor being located in the medial aspect of her right breast. What is one of the major concerns the doctor might have for this patient? A. metastasis to central nodes B. metastasis to parasternal nodes C. metastasis to apical nodes D. metastasis to subscapular nodes

B. metastasis to parasternal nodes

A gene encodes a protein with 150 amino acids. There is one intron of 1,000 bp, a 5'- untranslated region of 100 bp, and a 3'-untranslated region of 200 bp. In the final processed mRNA, how many bases lie between the start AUG codon (include the start codon) and the final termination codon? A) 1,750 B) 750 C) 650 D) 450 E) 150

D) 450

If a 1,000-kilobase fragment of DNA has 10 evenly spaced and symmetric replication origins and DNA polymerase moves at 1 kilobase per second, how many seconds will it take to produce two daughter molecules (ignore potential problems at the ends of this linear piece of DNA)? Assume that the 10 origins are evenly spaced from each other, but not from the ends of the chromosome. A) 20 B) 30 C) 40 D) 50 E) 100

D) 50

A mutation to which of the following nucleotide sequences in eukaryotic mRNA will affect the process by which the 3'-end poly-A tail is added to the mRNA. A) CAAT B) CCA C) GGGGCG D) AAUAAA E) TATAAA

D) AAUAAA

The sequence of part of a DNA strand is the following -ATTCGATTGCCCACGT-. When this strand is used as a template for DNA synthesis, the product will be which one of the following? A) TAAGCTAACGGGTGCA B) UAAGCUAACGGGUGCA C) ACGUGGGCAAUCGAAU D) ACGTGGGCAATCGAAT E) TGCACCCGTTAGCTTA

D) ACGTGGGCAATCGAAT

A herpetologist is bitten by a poisonous snake and is brought into the emergency room with progressive muscle paralysis. The venom is probably incapacitating his: A) Na+ channels B) Ca2+ channels C) Phospholipids D) Acetylcholine receptors E) Spectrin

D) Acetylcholine receptors

A hypothetical patient was suffering from excessive free iron in the blood, yet a cellular analysis indicated low intracellular levels of iron, despite high intracellular levels of ferritin, and normal transferrin levels in the blood. The disease was shown to be caused by a single base change in the DNA that led to a dysfunctional protein. The mutation is likely to be in which of the following proteins? A) Transferrin B) Transferrin receptor C) Transcobalamin D) Aconitase E) Ceruloplasmin

D) Aconitase

The stability and arrangement if actin filaments as well as their properties and functions depend on which one of the following? A) The structure of the actin filaments B) Microtubules C) Intermediate filament proteins D) Actin binding proteins E) Motor molecules, such as kinesin

D) Actin binding proteins

What kind of enzyme is telomerase? A) A DNA-dependent DNA polymerase B) A DNA-dependent RNA polymerase C) An RNA-dependent RNA polymerase D) An RNA-dependent DNA polymerase E) A protein-dependent DNA polymerase

D) An RNA-dependent DNA polymerase

The primary function of the nucleolus involves: A) Transfer of ribosomal RNA into the lumen of the nuclear envelope B) Synthesis of most, but not all, ribosomal proteins C) Packaging of mRNA and transfer RNAs into preribosomal subunits D) Assembly of several kinds of proteins, synthesized in the cytoplasm, with ribosomal RNA E) Pairing of daughter chromatids as the S-phase continues

D) Assembly of several kinds of proteins, synthesized in the cytoplasm, with ribosomal RNA

A building block of microtubules:

Beta-tubulin

You want to practice taking blood pressure on your friend. You remember you were taught to look for a tendon as a landmark to find the brachial artery. This tendon belongs to the ______________ muscle

Biceps brachii

In the cubital fossa, what is medial to the biceps brachii tendon?

Brachial artery

Upon returning home from your first semester in medical school, you decide to relieve some tension by going to the gym with a friend who has gotten into bodybuilding. He tells you that you have "sissy arms" and should work on your biceps, "the most important muscle for flexing the forearm". Being that you have gained so much anatomy knowledge, you decide to tell him that the biceps is in fact not the most important flexor of the forearm. Rather, it is what muscle?

Brachialis muscle

Which one of the following transport processes require energy? A) Facilitated diffusion B) Passive transport C) Active transport D) Simple diffusion

C) Active transport

The peptide bond formed between 2 adjacent amino acids is formed by what kind of catalytic reaction? A) An isomerization reaction B) An oxido-reductase reaction C) An acid-base reaction D) A Fenton reaction E) A Schiff-base reaction

C) An acid-base reaction

To what do the mRNA codons align? A) Anticodon on the mRNA B) Codon on the mRNA C) Anticodon on the tRNA D) Anticodon on the rRNA E) Codons on miRNA

C) Anticodon on the tRNA

Why does every DNA replication fork generate leading and lagging replication strands? A) Because helicase unwinds DNA in the 5' to 3' direction B) Because DNA replication required a double stranded template to begin DNA synthesis C) Because DNA is an antiparallel double helix D) Because DNA polymerase can synthesize DNA in both the 5' to 3' and 3' to 5' direction E) Because Okazaki fragments form on only one side of the replication bubble

C) Because DNA is an antiparallel double helix

The statement "DNA integrity is the top priority for all cells" means what? A) To help ensure that DNA is accurately replicated, it used RNA primers to specify where replication begins on the chromosome B) Cells expend more ATP energy on DNA synthesis than on gene transcription C) Cells have elaborate mechanisms to ensure that their DNA is not damaged during cell division D) Cells copy DNA in the 5' to 3' and 3' to 5' directions simultaneously to reduce the time of replication E) To protect the DNA, proteins that enter the nucleus must also exit the nucleus

C) Cells have elaborate mechanisms to ensure that their DNA is not damaged during cell division

A 2-year-old girl exhibits a very high fever of sudden onset and complains of a stiff neck. Physical exam reveals a positive Brudzinski and Kernig sign and petechiae on the extremities. The pediatrician, in addition to rushing the child to the hospital, prescribes a drug that blocks prokaryotic peptide bond formation, even though it can have serious side effects. That drug is which of the following? A) Rifampin B) Rapamycin C) Chloramphenicol D) Cycloheximide E) Puromycin

C) Chloramphenicol

A 14-year-old girl has had Type 1 diabetes since she was 10 years old. Recently she has been "forgetting" to take her insulin, and consequently she has had 3 episodes of diabetic ketoacidosis in the last 6 months. Her family physician meets with her and says, "Tell me what has gotten tough about taking your insulin on schedule." The patient states that she gains weight when she takes all of her insulin and she does not want to get fat. Which of the following best describes the patient-centered technique used by this physician? A) Demonstrating empathy B) Creating rapport C) Eliciting the patient's perspective D) Conflict resolution E) Reflection of meaning

C) Eliciting the patient's perspective

Which of the following statements is true about the regulation of ferritin and the transferrin receptor? A) Transferrin receptor has a regulatory element in its 5'UTR. B) Ferritin has a regulatory element in its 3'UTR. C) Ferritin has a regulatory element in its 5' UTR. D) Transferrin receptor expression is controlled at the level of transcription. E) Ferritin expression is controlled at the level of transcription.

C) Ferritin has a regulatory element in its 5' UTR.

During DNA replication, which type of enzyme separates double-stranded DNA into the single-stranded DNA that is needed as a template for the DNA polymerases? A) DNase B) Topoisomerase C) Helicase D) Primase E) Single-strand DNA binding (SSB) protein

C) Helicase

Clumps of nucleoprotein concentrated near the periphery of the nucleus: A) Nuclear pore complex B) Nucleolus C) Heterochromatin D) Outer nuclear membrane E) Euchromatin

C) Heterochromatin

Lipases cleave fatty acid chains from the glycerol backbone in phospholipids, breaking the ester bond in the process and creating a hydroxyl group on the glycerol and a carboxylic acid on the released fatty acid. To what class of enzymes do lipases belong? A) Oxidoreductase B) Transferase C) Hydrolase D) Lyase E) Isomerase F) Ligase

C) Hydrolase

The cell membrane consists of various components, including intergral proteins. These intergral proteins. A) Are not attached to the outer leaflet B) Are not attached to the inner leaflet C) Include transmembrane proteins D) Are preferentially attached to the E-face E) Function in the transport of cholesterol-based hormones

C) Include transmembrane proteins

The catalytic efficiency of two different enzymes can be compared by the A) formation of the product B) pH of optimum value C) Km value D) molecular size of the enzyme,

C) Km value

The genetic code is said to be degenerate because of which of the following? A) Many codons have pairs of identical bases next to each other. B) Some triplets are made up of repeating purines or pyrimidines. C) Many of the amino acids have more than one triplet codon. D) There is wobble in the bond between the first base of the anticodon and the third base of the codon. E) All triplets seem to have at least one uracil.

C) Many of the amino acids have more than one triplet codon.

Under conditions where methionine must be the first amino acid, what protein would be coded for by the following mRNA? 5'-CCUCAU AUG CGC CAU UAU AAG UGA CACACA-3' A) Pro-His-Met-Arg-His-Tyr-Lys-Cys-His-Thr B) Met-Arg-His-Tyr-Lys-Cys-His-Thr C) Met-Arg-His-Tyr-Lys D) Met-Pro-His-Met-Arg-His-Tyr-Lys-Cys-His-Thr E) Arg-His-Ser-Glu-Tyr-Tyr-Arg-Leu-Tyr-Ser

C) Met-Arg-His-Tyr-Lys

Has a rigid wall composed of 13 protofilament strands. A) Microfilament B) Intermediate filament C) Microtubule

C) Microtubule

Is associated with kinesin. A) Microfilament B) Intermediate filament C) Microtubule

C) Microtubule

Which one of the following substances is unable to transverse the plasma membrane by simple diffusion? A) O2 B) N2 C) Na+ D) Glycerol E) CO2

C) Na+

Which of the following statements are true about the genetic code? A) A given mRNA can code for many polypeptide chains B) Many triplets can be "nonsense" triplets C) No signal exists to indicate the end of one codon and the beginning of another D) The nucleotide on the 5' end of a codon triplet has the least specificity for an amino acid E) Gene sequence and encoded proteins are not colinear

C) No signal exists to indicate the end of one codon and the beginning of another

The nucleolus would be most metabolically active in which type of cell? A) One actively engaged in phagocytosis B) One actively engaged in steroid biosynthesis C) One actively engaged in protein synthesis D) One actively engaged in contraction E) An erythrocyte

C) One actively engaged in protein synthesis

Which of the following nucleotide sequences is never found in an exon? A) 5' UTR B) 3' UTR C) Promoter D) Introns E) Codons for amino acids

C) Promoter

What catalyzes the hydrolytic step leading to the separation of a polypeptide chain from a ribosome? A) Stop codons B) Peptidyl transferase C) Release factors D) Dissociation of ribosomes E) UAA

C) Release factors

A 30-year-old man who is vomiting, has bloody diarrhea, is dehydrated, and is delirious, is brought to the emergency department. The attending physician is told that he is an amateur chef who was trying out a new creation in which he wanted to experiment with the extracts of castor beans. This person's symptoms are all due to which of the following? A) Inhibition of RNA polymerase I B) Inhibition of RNA polymerase II C) Ribosomal inactivation by covalent modification D) Ribosomal disassembly due to covalent modification E) Inhibition of amino-acyl tRNA synthetases

C) Ribosomal inactivation by covalent modification

During which phase of the cell cycle is DNA replicated? A) G0 B) G1 C) S D) G2 E) M

C) S

A 54-year-old patient presents to the primary care clinic for a routine check-up. During the medical interview, the patient discloses that he is recently divorced, has gone on a number of dates, and has numerous sexual partners without using protection. The physician decides to test the patient for sexually transmitted illnesses (STI). Which of the following best describes the level of prevention activity exercised by this physician? A) Primordial B) Primary C) Secondary D) Tertiary E) Quaternary

C) Secondary

All of the following are integral membrane proteins of the erythrocyte membrane EXCEPT: A) Glycophorin B) Band 3 protein C) Spectrin D) Glucose translocase E) Na+/K+ ATPase

C) Spectrin

Examination of the blood smear of a young patient reveals mishappen red blood cells, and the pathology report indicates hereditary spherocytosis. Defects in which one of the following proteins causes this condition? A) Signaling molecules B) G proteins C) Spectrin D) Hemoglobin E) Ankyrin

C) Spectrin

What is a characteristic of a lipid raft? A) Lipid rafts are immobile in the membrane B) Lipids rafts are linked to the extracellular matrix C) Sphingolipids cluster in lipid rafts D) Cholesterol is excluded from lipid rafts E) Lipids rafts are linked to the cytoskeleton

C) Sphingolipids cluster in lipid rafts

The following four items are involved in the initiation of transcription. Which of the following four is composed of DNA? A) RNA polymerase B) Transcription factors C) TATA box D) Activators

C) TATA box

In eukaryotes, to what does cycloheximide bind to inhibit the peptidyl transferase reaction? A) The 50S ribosomal subunit B) Elongation factor Tu C) The 60S ribosomal subunit D) The 30S ribosomal subunit E) Elongation factor eEF-2

C) The 60S ribosomal subunit

What coordinates transcription with pre-mRNA processing? A) The newly made mRNA B) The ribosome C) The C-terminal domain of RNA pol II D) The N-terminal domain of RNA pol II E) The spliceosome

C) The C-terminal domain of RNA pol II

The 5' UTR contains which of the following? A) Splice sites B) Introns C) The mRNA cap D) Poly A tail E) The promoter sequence

C) The mRNA cap

Diphtheria toxin is produced by the gram positive bacilli Corynebacterium diphtheria. The toxin ribosylates eEF2. What is the effect of this modification? A) The peptide bond cannot be formed. B) The aminoacyl tRNA cannot enter the A site. C) The newly formed peptidyl tRNA cannot move to the P site. D) The initiator tRNA cannot enter into the P site. E) The empty tRNA in the E site cannot leave the ribosome.

C) The newly formed peptidyl tRNA cannot move to the P site

The removal of introns and subsequent self-splicing of adjacent exons occurs in some portions of primary ribosomal RNA transcripts. How does the splicing of introns in messenger RNA precursors occur? A) The processes are RNA-catalyzed in the absence of protein. B) The processes occur by self-splicing. C) The process is carried out by spliceosomes. D) The process is controlled by DNA polymerase. E) The process is regulated by RNA helicase.

C) The process is carried out by spliceosomes

A patient is shown to have a deletion of 500 base pairs within the 3.7 kilo-base-pair sequence between the promoter of the L-type pyruvate kinase (PK) gene and an upstream erythroid-cell-specific enhancer element for the PK gene. What is the most likely outcome of the mutation in erythroid cells? A) PK will be produced at a higher rate. B) Transcriptional initiation becomes impossible. C) There will be no change in the rate of enzyme synthesis. D) There will be faulty splicing of the transcript. E) The rate of transcription initiation by TFIIH will be reduced.

C) There will be no change in the rate of enzyme synthesis.

A genetic engineer announces the birth of a baby with an artificial chromosome that carries an extra copy of the gene for telomerase. The gene is engineered with an inducible promoter that causes transcription in response to the insect hormone ecdysone. Why should people with this artificial chromosome want to take ecdysone pills? A) To reduce the cancer risk B) To be resistant to viral infections C) To delay aging D) To get a high E) To cure cancer

C) To delay aging

Guanosine triphosphate (GTP) is required by which of the following steps in protein synthesis? A) Aminoacyl-tRNA synthetase activation of amino acids B) Attachment of ribosomes to endoplasmic reticulum C) Translocation of tRNA-nascent protein complex from A to P sites D) Attachment of mRNA to ribosomes E) Attachment of signal recognition protein to ribosomes

C) Translocation of tRNA-nascent protein complex from A to P sites

Cilia: What is the relevant cytoskeletal protein that forms or supports the structure. A) Actin B) Intermediate filament proteins C) Tubulin

C) Tubulin

Flagella: What is the relevant cytoskeletal protein that forms or supports the structure. A) Actin B) Intermediate filament proteins C) Tubulin

C) Tubulin

Microtubules: What is the relevant cytoskeletal protein that forms or supports the structure. A) Actin B) Intermediate filament proteins C) Tubulin

C) Tubulin

In addition to release factors (RFs), which of the following is needed to terminate translation? A) Rho B) A stem-loop structure C) UGA, UAG, or UAA codons D) AUG codons E) Consecutive trp codons

C) UGA, UAG, or UAA codons

Which one of the following nucleotides is present only in RNA? A) Thymine B) Adenine C) Uracil D) Cytosine E) Guanine

C) Uracil

Anticodons are located in: A) mRNA B) rRNA C) tRNA D) snRNP E) hnRNP

C) tRNA

Tom is a medical student in his surgery rotation. One day Dr. Heytst Udens, the surgeon, asked his students to put these in order from deep to superficial and anterior to posterior: 1. Posterior internal vertebral venous plexus 2. Anterior external vertebral venous plexus 3. Anterior internal vertebral venous plexus 4. Posterior external vertebral venous plexus A. 1-2-3-4 B. 4-3-2-1 C. 2-3-1-4 D. 2-3-4-1 E. 4-1-3-2 F. 1-4-3-2

C. 2-3-1-4

Use the Henderson Hasselbach equation to determine the pH of a solution containing 5 mM acetic acid [AH] and 10 mM sodium acetate [A-]. The pK of acetic acid is 4.75. A. 4.0 B. 4.45 C. 5.05 D. 5.55 E. 6.0

C. 5.05

Imidazole is the name of the side chain on histidine, and can be used as a buffer by itself. It has a pK of 7.2. At what pH will this be a good buffer: A. 5 B. 6 C. 7 D. 8 E. 9

C. 7

The component of a water-soluble globular protein that is most likely to be present in the center of the molecule rather than on its surface is A. A glutamate side chain B. A histidine side chain C. A phenylalanine side chain D. A phosphate group covalently linked to a serine side chain E. An oligosaccharide covalently linked to an asparagine side chain

C. A phenylalanine side chain

Diane has a breast abscess. What is the most superior lymph node in her axilla which will drain the node? A. Supraclavicular lymph nodes B. Central lymph nodes C. Apical lymph nodes D. Superior mammary lymph nodes E. Superior pectoral lymph nodes

C. Apical lymph nodes

A 21 year old patient has a lesion of the upper trunk of the brachial plexus (Erb Duchenne paralysis). Which of the following is the most likely diagnosis? A. Paralysis of the rhomboid major B. Inability to elevate the arm above the horizontal C. Arm tending to lie in medial rotation D. Loss of sensation on the medial side of the arm E. Inability to adduct the thumb

C. Arm tending to lie in medial rotation

A 22-year-old male presents to your office complaining of pain and loss of function in his right arm. The patient tells you he was out horse back riding when he was thrown from the horse and landed hard on his right arm. Upon inspection you notice that the patient's right arm is medially rotated and there is a large area of swelling near the head of the humerus. The patient is unable to initiate abduction of the arm, but can abduct the arm to 90 degrees with help. With flexed elbows, he is unable to laterally rotate his arm against resistance. Based on the history and physical examination, which of the following is a possible diagnosis? A. Damaged axillary nerve B. Avulsion fracture of the lesser tubercle C. Avulsion fracture of the greater tubercle D. Dislocation of the head of the humerus E. Avulsion fracture of the deltoid tuberosity

C. Avulsion fracture of the greater tubercle

A 50-year-old woman reports to her physician after discovering a lump in her superolateral quadrant of her left breast. Her physician performs a breast exam, and notices enlarged lymph nodes in the process. Which group of lymph nodes are most likely to be enlarged in this case initially? A. Parasternal B. Subdiaphragmatic C. Axillary D. Subclavian

C. Axillary

If the middle trunk was severed, which nerve would not be affected? A. Thoracodorsal nerve B. Radial nerve C. Axillary nerve D. Median nerve

C. Axillary nerve

A nerve that travels in your upper arm is damaged. As a result, you experience weakness when you flex your forearm from a supinated position. What muscle is affected as a result of the damage? A. Brachioradialis muscle B. Brachialis muscle C. Biceps brachii muscle D. Coracobrachialis muscle

C. Biceps brachii muscle

A patient was admitted to the ER with a deep laceration in the cubital fossa area. Upon examination, the wound was found to be just medial to the biceps brachii tendon. Based on this finding, which of the following structures is most likely to have been injured? A. Cephalic vein B. Radial artery C. Brachial artery D. Radial nerve E. Ulnar nerve

C. Brachial artery

. A 25-year old female is hit on the side of her forearm and the muscles which make up the floor of the cubital fossa are torn. Which of the following groups of muscles have lost their function? A. Pronator teres muscle and supinator muscle B. Brachialis muscle and pronator teres muscle C. Brachialis muscle and supinator muscle D. Supinator muscle and brachioradialis muscle E. Biceps brachii muscle and brachialis muscle

C. Brachialis muscle and supinator muscle

When a patient undergoes a modified radical mastectomy, which of these structures is not removed? A. Breast Tissue B. Lymph Nodes C. Breast Muscle

C. Breast Muscle

A high school football player suffers a herniated intervertebral disk in his neck which compresses the spinal nerve exiting between the third and fourth cervical vertebrae. Which nerve is affected? A. C2 B. C3 C. C4 D. C5 E. C6

C. C4

A 46-year-old woman was diagnosed with advanced breast cancer and underwent mastectomy. After recovery from the surgery, she noticed that she could not raise her arm or wash her hair. Upon physical examination the physician noticed a protrusion of the scapula from its position. What are the roots of the nerve that most likely got damaged during the surgery? A. C5 B. C5, C6 C. C5, C6, C7 D. C6, C7,C8 E. C5, C6, C7, C8, T1

C. C5, C6, C7

A 20 year-old male presents to the ER with a stab wound to his left 3rd intercostal space near the mid-axillary line. Radiograph reveals a hemothorax (blood in chest cavity) so he is treated with a chest-tube insertion and admitted to critical care. He recovers, but when he pushes against a wall with outstretched arms, his left scapula protrudes dorsally. What are the roots of the damaged nerve? A. C3-C5 B. C6-C7 C. C5-C7 D. C6-C8 E. C5-T1

C. C5-C7

The nerve found in the triangular interval receives its roots from which spinal nerves? A. C5-C6 B. C8-T1 C. C5-T1 D. C6-C8

C. C5-T1

While mountain biking at Tom Brown park, your friend Jeff took a nasty fall. You went to help him up and make sure he was okay. Other than a few scrapes on his legs and arms everything seemed okay, but when he tried to grab his handle bars you noticed he could not completely flex his last two fingers. Attempt to make a fist results in a typical "claw hand" presentation. Which nerve roots did Jeff injure on his bicycle accident? A. C5-C6 B. C8 C. C8-T1 D. C7-T1 E. C8-T2

C. C8-T1

In a bar fight, a guy gets a cut in his shoulder/arm region, at the deltopectoral groove, his bleeding is most likely from which of these vessels? A. Axillary artery B. Basilic vein C. Cephalic vein D. Brachial profunda artery

C. Cephalic vein

Identify which can be found in the Triangular Space. A. Radial nerve and deep artery of the arm B. Axillary nerve and Posterior circumflex humeral artery C. Circumflex Scapular Artery D. Suprascapular nerve

C. Circumflex Scapular Artery

What artery exists in the triangular space? A. Posterior circumflex numeral B. Thoracodorsal C. Circumflex scapular D. Anterior circumflex numeral

C. Circumflex scapular

The cephalic vein pierces what fascia to drain into which vein? A. Pectoral fascia, axillary vein B. Pectoral fascia, subclavian vein C. Clavipectoral fascia, axillary vein D. Clavipectoral fascia, subclavian vein

C. Clavipectoral fascia, axillary vein

You recently examine a patient with weakened flexion of the arm and parasthesia on the lateral aspect of the forearm. Hypertrophy of which muscle would most likely cause this clinical presentation? A. Pectoralis Minor B. Long head of the Triceps C. Coracobrachialis D. Brachialis E. Supinator

C. Coracobrachialis

A man presents to the emergency department with severe shoulder pain. The attending physician learns that he was standing on a ladder cleaning out the gutters of his house. He tells the physician that he had leaned too far to the side and the ladder tipped over, and he fell to the ground, landing on his right shoulder. Upon physical examination, the physician finds that the medial portion of the man's clavicle is displaced. How is this portion of the bone positioned in this type of injury, and what structure is causing this displacement? A. Elevated by Trapezius B. Depressed by Pectoralis major C. Elevated by Sternocleidomastoid D. Depressed by the weight of the arm

C. Elevated by Sternocleidomastoid

An ice skater slips and falls directly on his shoulder injuring his superior trunk of his right brachial plexus. Which of the following actions would NOT be compromised? A. Abduction of the humerus B. Adduction of the humerus C. Inferior rotation of scapula D. Lateral rotation of the humerus E. Medial rotation of the humerus

C. Inferior rotation of scapula

Your grandmother has become increasingly forgetful during the past 2 years. Last week she actually got lost on the way back from the grocery store a few blocks down the road. One treatment that could perhaps help her would be a drug that A. Reduces the synthesis of transthyretin B. Reduces the formation of immunoglobulins C. Inhibits the activity of β-secretase D. Reduces the formation of α-synuclein E. Adds phosphate groups to tau protein

C. Inhibits the activity of β-secretase

A couple of hours after your normal breakfast of oatmeal, and sitting in class, you go to you next class. On the way, you stop at McDonalds and order a snack of a supersized Soda (nondiet) and fries and promptly eat them while walking slowly to your next class. What is the main fate of the starch in your fries, while you are sitting in class? A. It will be used for energy. B. It will be stored as glycogen. C. It will be converted to fat. D. It will stored in skeletal muscle for future use. E. It will be converted to protein.

C. It will be converted to fat.

Which is true of the anterior ramus of the C7 spinal nerve? A. Its fibers are purely sensory and originate between C6 and C7 vertebrae B. Its fibers are purely motor and originate between C7 and T1 vertebrae C. Its fibers contain both motor and sensory and originate between C6 and C7 vertebrae D. Its fibers contain both motor and sensory and originate between C7 and T1 vertebrae

C. Its fibers contain both motor and sensory and originate between C6 and C7 vertebrae

A post-fixed brachial plexus increases the risk for which brachial neuropathy? A. Wrist drop B. Hand of benediction C. Klumpke's palsy D. Erb's palsy E. Ape hand

C. Klumpke's palsy

A patient comes into your office with the chief complaint of back pain. Upon further examination you determine that they are suffering from a herniation of the L3/L4 intervertebral disc. Knowing this you can correctly diagnose which spinal nerve is being compressed, this nerve is: A. L3 B. L5 C. L4 D. L2

C. L4

What is the main product of pyruvate during anaerobic metabolism in muscle? A. Ethanol B. AcetylCoA C. Lactate D. Oxaloacetate

C. Lactate

Lateral pectoral nerve arises from part of the brachial plexus? A. Middle Trunk B. Posterior Cord C. Lateral Cord D. Superior Trunk E. Medial Cord

C. Lateral Cord

Lateral pectoral nerve arises from what part of brachial plexus? A. Medial Cord B. Middle trunk C. Lateral Cord D. Posterior Cord E. Superior Trunk

C. Lateral Cord

A cricket player dislocated his shoulder during a game. The displaced humeral head came to rest anterior to the glenohumeral joint, inferior to the coracoid process. Given the most likely nerve injury with this type of dislocation, where would you expect paresthesia in this patient? A. Posterior shoulder B. Anterior shoulder C. Lateral shoulder D. Posterior arm and forearm E. Medial arm and forearm

C. Lateral shoulder

A cricket player dislocated his shoulder during a game. The displaced humeral head came to rest anterior to the glenohumeral joint, inferior to the coracoid process. Given the most likely nerve injury with this type of dislocation, where would you expect paresthesia in this patient? A. Posterior Shoulder B. Anterior shoulder C. Lateral shoulder D. Posterior arm and forearm E. Medial arm and forearm

C. Lateral shoulder

A patient presents with peau d'orange sign on her left breast. The cancer has most likely invaded what structure/space? A. Suspensory ligaments B. Mammary glands C. Lymph vessels D. Retromammary space E. Lactiferous duct

C. Lymph vessels

A 57 year-old woman presents with retraction and deviation of her right nipple. Where is the most likely invasion of cancer in the breast? A. Glandular tissue B. Lactiferous ducts C. Lymphatic drainage D. Retromammary space

C. Lymphatic drainage

A 55 year old woman goes in for a routine physical. The doctor notices that there is an abnormality in the breast. He notices an orange peel appearance and suspects breast cancer. If this diagnosis proves to be correct, where has the cancer likely spread to? A. Lactiferous Duct B. Glandular Tissue C. Lymphatics D. Retromammary space E. Pectoral Fascia

C. Lymphatics

Your patient Stephen Huntley, a 42 year old male, was in a car accident. His right distal humerus was crushed in the crash. He's in extreme pain and has weak pronation and flexion in his right forearm, wrist and hand. What nerve is most likely implicated in this situation? A. Axillary B. Radial C. Median D. Ulnar

C. Median

A nervous 3rd year medical student was both excited and scared to be on his first rotation in Internal medicine. The student's attending physician asked him to place a simple IV catheter in a patients right median cubital vein. With a shaky hand the student inserted the catheter, confident that the bicipital aponeurosis would prevent him from damaging what underlying structure? A. Ulnar nerve B. Radial nerve C. Median nerve D. Cephalic vein E. Basilic vein

C. Median nerve

A patient comes into your office complaining of weakness in his arm. The patient tells you that he often goes to the gym and enjoys heavy weight lifting. This past weekend he was with his buddies and they decided to have an arm wrestling contest. Your patient won the contest but he has not been the same since. He tells you that when he goes to the gym now he is unable to "curl" or flex his arm when lifting the dumbbells. He wants to know what is wrong so you perform a physical exam and determine the cause is of neurovascular origin. Which nerve is most likely damaged? A. Radial B. Median C. Musculocutaneous D. Axillary

C. Musculocutaneous

Your 38 year old patient, Harvey Spector was rushed into the emergency room for a penetrating injury to his arm. After he was treated for his injuries and give proper pain medication, you are asked by your attending to test for any motor or sensory defects. You find that the biceps brachii is affected. What nerve is most likely injured? A. Axillary (C5,C6) B. Radial (C5-C8, T1) C. Musculocutaneous (C5-C7) D. Ulnar (C7-T1)

C. Musculocutaneous (C5-C7)

Which of the following molecule types is not a major energy store in the human body? A. Fat (triglycerides) B. Protein C. Nucleotides (DNA and RNA) D. Glycogen

C. Nucleotides (DNA and RNA)

A 47 year old female patient is diagnosed with carcinoma of the breast. During the physical examination, you palpate an enlarged mass in the left axillary region, a finding which leads you to suspect that the cancer has spread through lymphogenic metastasis. What lymph node is most likely enlarged? A. Humeral B. Subscapular C. Pectoral D. Parasternal E. Infraclavicular

C. Pectoral

A 25-year-old male suffers a shoulder injury during a football game and comes into the emergency department. X-rays show a fractured coracoid process of the right scapula. Which muscle will MOST likely be affected? A. Supraspinatus B. Pectoralis Major C. Pectoralis Minor D. Brachialis

C. Pectoralis Minor

A 72-year-old man with Type 2 Diabetes, well controlled with diet and medication, has been doing well during his visits with his doctor every 3-6 months. Today he complains to his physician that his life seems senseless and that he ought to be "put down, like an old sick dog". Using a relationship-centered approach, which of the following is the most important next step for the physician working with this patient? A) Prescribe antidepressant medication B) Refer patient for psychotherapy C) Elicit family history of depression, anxiety and psychosis D) Elicit the patient's perspective and reason for the change in his feelings E) Set up a "no harm" contract as a suicide precaution

D) Elicit the patient's perspective and reason for the change in his feelings

A one-year-old boy with chronic anemia is found to have beta-thalassemia. Genetic analysis shows that one of the beta-globin genes has a G to A mutation that creates a new splice acceptor site nineteen nucleotides upstream from the normal splice acceptor site of the first intron. Which of the following best describes the new messenger RNA molecule that can be produced from this mutant gene? A) Exon 1 will be too short. B) Exon 1 will be too long. C) Exon 2 will be too short. D) Exon 2 will be too long. E) Exon 2 will be missing.

D) Exon 2 will be too long.

Which of the following does not have an ATP or GTP requirement? A) Attachment of an amino acid to its tRNA B) Placement of a new aminoacyl-tRNA on the ribosome during elongation C) Movement of the mRNA through the ribosome D) Formation of the peptide bond during protein synthesis E) Assembly of the translational initiation complex

D) Formation of the peptide bond during protein synthesis

Adrenocorticotropic hormone (ACTH) travels through the bloodstream, enters connective tissue spaces, and attaches to specific sites on target-cell membranes. These sites are: A) Peripheral proteins B) Signaling molecules C) G proteins D) G-protein-linked receptors E) Ribophorins

D) G-protein-linked receptors

A 49-year-old man has asthma which is exacerbated when he uses tobacco. He visits his physician because he wants to quit smoking. He has tried unsuccessfully to quit multiple times in the past. Which of the following is the best relationship-centered statement for the physician to make in counseling this patient? A) My husband used to smoke and he still craves nicotine a lot of the time. B) There's a new kind of patch I'd like to try you on if you're game. C) If you don't get your smoking under control, it will kill you. D) I know how difficult this is and am glad you're willing to try again. E) Many of my patients have found a behavioral contract helpful.

D) I know how difficult this is and am glad you're willing to try again.

A 16-year-old girl has been losing weight and feeling lethargic over the past 4 months and is taken to the physician by her parents. During the history, the parents expressed concern that their daughter has seemed to eat very little during the day, a claim denied by the patient. Laboratory results indicated an iron deficiency and a microcytic anemia. The cells of the patient have adapted to the iron deficiency in which one of the following ways? A) Increased transcription of ferritin mRNA B) Reduced transcription of the transferrin receptor mRNA C) Increased translation of the ferritin mRNA D) Increased translation of the transferrin receptor mRNA E) Increased degradation of the transferrin receptor mRNA

D) Increased translation of the transferrin receptor mRNA

In a typical eukaryotic cell the nucleus: A) Has a diameter of between 50 and 250 micrometers B) Is tightly connected to the plasma membrane by bundles of microtubules C) Contains chromatin, which is a complex of acidic proteins and glycolipids D) Is surrounded by a double membrane, the outer later of which has ribosomes bound to it E) Is the principle site of protein synthesis

D) Is surrounded by a double membrane, the outer later of which has ribosomes bound to it

Pyruvate Decarboxylase is an integral activity of the pyruvate dehydrogenase complex, but carries out a discrete reaction: the removal of CO2 from pyruvate. Which class of enzymes does this particular activity belong to? A) Oxidoreductase B) Transferase C) Hydrolase D) Lyase E) Isomerase F) Ligase

D) Lyase

A male child at puberty is determined to have Klinefelter syndrome. Although the parents have been informed of the clinical significance, they have asked for an explanation of what happened. Identify the item that needs to be discussed with the parents. A) Trisomy of chromosome 21 B) Loss of an autosome during mitosis C) Loss of the Y chromosome during meiosis D) Nondisjunction of the X chromosome

D) Nondisjunction of the X chromosome

Continuous with rough ER: A) Nuclear pore complex B) Nucleolus C) Heterochromatin D) Outer nuclear membrane E) Euchromatin

D) Outer nuclear membrane

The nuclear pore complex: A) Permits free communication between the nucleus and the cytoplasm B) Is bridged by a unit membrane C) Is located only at specific nuclear pore sites D) Permits passage of proteins via receptor-mediated transport

D) Permits passage of proteins via receptor-mediated transport

What does a stop codon interact with that causes translation to cease? A) Initiation factors B) Elongation factors C) A tRNA that has an anticodon for a stop codon D) Release factors E) One of the proteins in the small ribosomal subunit

D) Release factors

The form of chromatin that is called euchromatin: A) Is the most condensed form of chromatin in the nucleus B) Comprises the core of all chromosomes found at the onset of mitosis C) Usually is found as a thin layer tightly attached to the inner surface of the inner membrane of the nuclear envelope. D) Represents portions of the chromatin that are relatively decondensed and are sites where transcription into RNA can occur E) Can be distinguished from heterochromatin because euchromatin has very few exons as compared to heterochromatin

D) Represents portions of the chromatin that are relatively decondensed and are sites where transcription into RNA can occur

The membrane of the rough ER is continuous with: A) Peroxisomal membranes B) The inner membrane of mitochondria C) Mitochondrial outer membrane D) The outer membrane of the nuclear envelope E) The plasma membrane

D) The outer membrane of the nuclear envelope

The 3' UTR contains which of the following nucleotide sequences? A) The translational control element for ferritin B) The promoter sequence C) The mRNA cap nucleotides D) The polyadenylation sequence E) Splice donor sequence

D) The polyadenylation sequence

The codon UAA is referred to as a stop codon. What exactly is a stop codon? A) The stop codon instructs the transcription machinery to stop the process of transcription. B) The stop codon instructs the splicing machinery to stop the process of RNA splicing. C) The stop codon instructs the polyadenylation machinery to stop the process of adding As to the end of a transcript. D) The stop codon instructs the translation machinery to stop the process of translation. E) None of the above statements are true.

D) The stop codon instructs the translation machinery to stop the process of translation.

The reason there are 64 possible codons is which of the follow? A) There are 64 aminoacyl tRNA synthetases. B) Each base is able to participate in wobbling. C) All possible reading frames can be used this way. D) There are four possible bases at each of three codon positions. E) The more codons, the faster protein synthesis can be accomplished.

D) There are four possible bases at each of three codon positions.

Which of the following enables all DNA of a eukaryotic cell to be replicated in only 8 hours? A) Eukaryotic DNA polymerases are exceptionally fast. B) The β-sliding clamp (PCNA) enables the polymerase to stay on the DNA. C) There are no single-stranded binding proteins to slow down the polymerases. D) There are multiple origins of replication. E) The eukaryotic helicases open the DNA very rapidly.

D) There are multiple origins of replication.

Which of the following are true about the so-called caps of RNA molecules? A) They allow tRNA to be processed. B) They occur at the 3' end of tRNA. C) They are composed of poly A. D) They are necessary for the ribosome to find the 5' end of the mRNA. E) The nucleotides the make up the cap are encoded in the DNA.

D) They are necessary for the ribosome to find the 5' end of the mRNA.

What is a characteristic of peripheral membrane proteins of the plasma membrane? A) They are immobile in the membrane B) They can be found only on the cytoplasmic face of the plasma membrane C) They can be covalently linked to the membrane by phospholipids D) They can be removed from the membrane with high salt E) They are held at the membrane via hydrophobic interactions with membrane components

D) They can be removed from the membrane with high salt

The extent of DNA synthesis in a cell could most specifically be determined by measuring the incorporation of which of the following radiolabeled compounds? A) Leucine B) Phosphate C) Ribose D) Thymidine E) Uracil

D) Thymidine

When the polio virus enters a cell, it takes over the cell's translational machinery by cleaving which of the following? A) eIF2 B) eIF4A C) eIF4E D) eIF4G E) eIF5

D) eIF4G

Symport refers to the process of transporting: A) a molecule into the cell B) a molecule out of the cell C) two different molecules in the opposite direction D) two different molecules in the same direction E) a molecule between the cytoplasm and the nucleus

D) two different molecules in the same direction

The following structure is an oligopeptide that is acetylated at its amino end and amidated at its carboxyl end, making the terminal groups nonionizable. This oligopeptide has a pI close to Acetyl-Ala-Glu-His-Ser-Lys-Gly-amide A. 4.3 B. 5.1 C. 6.0 D. 7.5 E. 10.8

D. 7.5

You measure the enzyme kinetics of a novel enzyme and you find the velocity at 3 mM to be 25% of the Vmax. What is the Km of this enzyme? A. 0.75 mM B. 3 mM C. 6 mM D. 9 mM E. 12 mM F. 20 mM

D. 9 mM

A form of hexokinase has a Km of about 500 micromolar for glucose. If the intracellular glucose is 5 mM, what will the approximate velocity of this isozyme be relative to the Vmax? A. ¼ Vmax B. ½ Vmax C. ¾ Vmax D. 90 % Vmax E. Vmax

D. 90 % Vmax

In the above vignette which of the following constitutes the history of the present illness (HPI): A. A thin white male in no apparent distress with a normal cardiac and respiratory examination. B. An appendectomy at age 25 because of a ruptured appendix and sepsis C. A hip replacement at age 76 because of severe osteoarthritis D. A patient with hypertension, diabetes and osteoarthritis presenting with pain in the left shoulder for 4 days that is getting worse E. Currently elevated blood glucose on laboratory testing.

D. A patient with hypertension, diabetes and osteoarthritis presenting with pain in the left shoulder for 4 days that is getting worse

During a prison fight, a guard is stabbed in an area on his shoulder bordered by his teres minor, teres major, triceps brachii long head and the shaft of his humerus. A nerve was severed. After the guard has healed, what action of his arm is still most likely compromised? A. Lateral rotation B. Adduction C. Medial rotation D. Abduction E. Pronation

D. Abduction

The physician informs you that you have painful arc syndrome because of repeated raising and lowering of your arm. You know that painful arc syndrome is caused by inflamed tendons rubbing against what? A. Scapular spine B. Coracoid process C. Coronoid process D. Acromion process E. Suprascapular notch

D. Acromion process

A genetic engineer wants to produce athletes with increased hemoglobin concentration in the erythrocytes, to improve oxygen supply to the muscles. To do so, the water solubility of the hemoglobin molecule must be increased. Which of the following amino acid changes on the surface of the hemoglobin molecule is most likely to increase its water solubility? A. Arg → Lys B. Leu → Phe C. Gln → Ser D. Ala → Asn E. Ser → Ala

D. Ala → Asn

The prognosis in breast cancer is poorer as more proximal lymph nodes are found to have cancerous cells in them. Spread of cancer to which of the following axillary nodes would indicate the worst prognosis? A. Pectoral B. Lateral C. Central D. Apical E. Subscapular

D. Apical

During the FIFA World Cup finals, a drunk surgeon got in a fight with a drunk, shirtless, professional bodybuilder. Because of the well defined muscles on the back of the shirtless bodybuilder, the surgeon knew that he could take out his emergency scalpel and lacerate the ___________ in the ___________ which would impact ____________ muscle(s) of the bodybuilder. A. Long Thoracic nerve, Quadrangular space, Serratus Anterior B. Thoracodorsal nerve, Triangular space, Triceps and biceps C. Axillary nerve, Triangular space, Deltoid and Teres Minor D. Axillary nerve, Quadrangular Space, Deltoid and Teres Minor E. Axillary nerve, Quadrangular Space, Deltoid and Teres Major

D. Axillary nerve, Quadrangular Space, Deltoid and Teres Minor

Porins contain what type of characteristic domain structure? A. Transmembrane alpha helix B. 7-transmembrane helical structure C. IgG fold D. Beta-barrel E. Globin fold

D. Beta-barrel

Human blood plasma contains about 7% protein. These plasma proteins have pK values close to 4 or 5. In the test tube, these proteins will form an insoluble precipitate after all of the following treatments except A. Boiling the serum for 5 minutes B. Adding sodium chloride to a concentration of 35% C. Adjusting the pH to 4.5 D. Boiling the serum with 6 N hydrochloric acid for 10 hours E. Mixing one volume of plasma with two volumes of pure alcohol

D. Boiling the serum with 6 N hydrochloric acid for 10 hours

While performing phlebotomy on a patient, a nurse accidentally injures the median nerve in the cubital fossa. Which action(s) may the patient have difficulty performing? A. Flexion of the thumb B. Opposition of the thumb C. Extension of the thumb D. Both A and B E. Both A and C

D. Both A and B

Which of the following muscles is paired with the nerve that innervates it? A. Coracobrachialis Muscle - Radial Nerve B. Triceps Brachii - Musculocutaneous Nerve C. Biceps Brachii - Median Nerve D. Brachialis - Musculocutaneous Nerve E. Brachioradialis - Ulnar Nerve

D. Brachialis - Musculocutaneous Nerve

A man has injured his coracoid process and is unable to use the muscles that originate from there. Which of the following muscles will still permit flexion of his forearm? A. Biceps brachii short and long heads B. Brachialis and coracobrachialis C. Triceps and brachialis D. Brachialis, brachioradialis, and biceps brachii long head E. Triceps and biceps brachii long head

D. Brachialis, brachioradialis, and biceps brachii long head

Which muscle innervated by the radial nerve acts to flex a joint? A. Flexor Carpi Radialis Muscle B. Supinator Muscle C. Biceps Brachii Muscle D. Brachioradialis Muscle E. Long head, Triceps Brachii Muscle

D. Brachioradialis Muscle

What two muscles make up the medial and lateral borders of the cubital fossa? A. Pronator teres and biceps brachii B. Brachioradialis and flexor digitorum superficialis C. Biceps brachii and brachioradialis D. Brachioradialis and pronator teres E. Brachialis and pronator teres

D. Brachioradialis and pronator teres

What makes up the lateral border of the cubital fossa? A. Radius bone B. Pronator teres muscle C. Supinator muscle D. Brachioradialis muscle E. Bicipital aponeurosis

D. Brachioradialis muscle

Patient comes into the emergency room because of some pain in his arm and he is unable to make a fist. He tells you that he was on a ladder painting the windows in his home and for some reason the ladder slipped but he managed to grab hold of a handle with his right hand. Now when he tries to make a fist and cannot because his digit 4 and digit 5 do not flex and has lost sensation to this area as well. Knowing that this injury damaged a nerve which nerve roots are most likely damaged? A. C5 and C6 B. C6 and C7 C. C7 and C8 D. C8 and T1

D. C8 and T1

A patient comes into the ER with an elbow injury. The radiologist tells you that the medial epicondyle is fractured. Upon examination the patient presents with "clawing" of the fingers when asked to make a fist. What are the roots of the nerve that is most likely affected? A. C5-C6 B. C5-C7 C. C5-T1 D. C8-T1

D. C8-T1

A 40-year-old female patient with a significant family history presents with dimpling of the skin of the left breast. As her physician, you order a series of tests including a chest X-ray because you know that dimpling of the left breast is a sign that: A. Cancer has entered the retromammary space. B. Cancer is interfering with lymph drainage. C. Cancer has invaded the lactiferous ducts. D. Cancer has invaded the glandular tissue.

D. Cancer has invaded the glandular tissue.

A weight trainer comes to your office complaining of difficulty in flexing his right arm. He recalls hearing a snap after trying to lift a 150lbs dumbbell. You decide to send him to surgery. During the operation, you find that the long head of the biceps brachii is intact and immediately suspect damage to the short head of the biceps brachii instead. Which of the following locations would help you in assessing the short head of the biceps brachii? A. Tuberosity of ulna B. Posterior part of humerus C. Coronoid process D. Coracoid process E. Supraglenoid tubercle

D. Coracoid process

Which amino acid(s) are sulfur containing? A. Just Cysteine B. Serine and Threonine C. Methionine and Threonine D. Cysteine and Methionine E. Arginine and Lysine

D. Cysteine and Methionine

The accompanying drawing of an electron micrograph shows a section through an adipocyte (fat cell). What is a characteristic of the blue space marked with an asterisk? A) It contains a buildup of waste products from the lysosome B) In a living cell, it is filled with protein C) This is the nucleus D) This space is usually pigmented E) It contains material formed by the smooth endoplasmic reticulum

E) It contains material formed by the smooth endoplasmic reticulum

What enzyme is necessary to seal the phosphodiester backbone and fuse Okazaki fragments together? A) DNA pol δ B) DNA pol ε C) DNA pol α D) Helicase E) Ligase

E) Ligase

Chromatin does NOT make up all or part of what structure? A) Nucleolus B) 30 nm fiber C) Telomere D) Euchromatin E) Nuclear envelope

E) Nuclear envelope

The peptide bond formed between 2 adjacent amino acids is formed by what? A) A protein enzyme called an amino peptidyl transferase forms the bond. B) It is formed spontaneously without the aid of an enzyme. C) One of the proteins of the large ribosomal subunit is the catalyst. D) The 18 S rRNA is the catalyst for this reaction. E) The 28 S rRNA is the catalyst for this reaction.

E) The 28 S rRNA is the catalyst for this reaction.

The isolation of nascent Okazaki fragments during DNA replication led to the surprising discovery of uracil in the fragment. The uracil is present due to which of the following? A) Deamination of cytosine B) Chemical modification of thymine C) An error in DNA polymerase D) Failure of mismatch repair E) The need for a primer

E) The need for a primer

A 40-year-old woman has been diagnosed with pre-diabetes. She states that she hopes there is something she can do to avoid taking medication. Her BMI is 30. Which of the following is the most appropriate next response by the physician? A) You know, you really need to lose some weight. B) Tell me what you eat on a daily basis. C) You'll have to start exercising right away and don't give up. D) It's going to be hard to change. E) What do you know about the relationship between prediabetes and body weight?

E) What do you know about the relationship between prediabetes and body weight?

Which term is most accurately used to characterize the interior of the lipid bilayer of the plasma membrane? A) hydrophilic B) acidophilic C) basophilic D) amphipathic E) hydrophobic

E) hydrophobic

Of what is the peptidyl transferase enzyme composed? A) Protein B) DNA C) tRNA D) mRNA E) rRNA

E) rRNA

Enzymes are evolutionarily adapted to their environment. The company Cetus first patented the isolated and cloned DNA polymerase from a bacteria Thermus aquaticus for use in the polymerase chain reaction. This bacteria lives preferentially in hydrothermal vents or hot springs. What is the likely temperature optimum of this enzyme A. 10C B. 25C C. 37C D. 50C E. 80C

E. 80C

Two important concepts to understand are Dermatomes and Cutaneous innervation. Being able to differentiate the two can help with diagnosis of neurological disorders. What are the supplying dermatomes & cutaneous innervation of the ring finger (Digit IV)? A. C6/C7 & Median B. C6/C7 & Median/Radial C. C7 & Radial/Ulnar D. C7 & Radial/Ulnar/median E. C7/C8 & Radial/Ulnar/Median F. C7/C8 & Radian/Ulnar G. C8 & Radial/Median H. C8 & Radial/Ulnar/Median

E. C7/C8 & Radial/Ulnar/Median

In the above vignette which of the following would be placed in the review of systems (ROS): A. Diabetes B. Hypertension C. Osteoarthritis D. Pain in the left shoulder E. Constipation

E. Constipation

A 23 year-old male presents to the ER with a painful right shoulder following a sports injury. X-ray shows a fracture to the scapula extending to the base of the coracoid process. Which muscles are mostly affected by this kind of fracture by losing their attachments to this structure? A. Coracobrachialis, Short head of biceps, Pectoralis minor B. Short head of biceps, Teres minor, Sternal head of pectoralis major C. Coracobrachialis, Short head of biceps, Teres minor D. Short head of biceps, Long head of triceps, Teres minor E. Coracobrachialis, Short head of biceps, Pectoralis minor

E. Coracobrachialis, Short head of biceps, Pectoralis minor

What two muscles are innervated by the axillary nerve? A. Teres Major and Deltoid B. Supraspinatus and Infraspinatus C. Subscapularis and Teres Minor D. Trapezius and Supraspinatus E. Deltoid and Teres Minor

E. Deltoid and Teres Minor

Which of the following can use only glucose as a fuel source? A. Brain B. Muscle C. Kidney D. Adipose E. Erythrocytes

E. Erythrocytes

A 30 year old patient is unable to draw his scapula forward and downward because of paralysis of the pectoralis minor. Which of the following would most likely be a cause of his condition? A. Defect in the posterior wall of the axilla B. Axillary nerve injury C. Fracture of the clavicle D. Injury of the posterior cord of the brachial plexus E. Fracture of the coracoid process

E. Fracture of the coracoid process

In the above vignette, which of the following would most appropriately be placed in his previous medical history (PMH)? A. Pain in his left shoulder B. Currently elevated blood glucose C. Normal respiratory exam D. Normal cardiac exam E. Hip replacement at age 70

E. Hip replacement at age 70

Most secondary structures form to satisfy principally this type of interaction: A. Hydrophobic Interactions B. Van der Waals forces C. Ionic Bonding D. Disulfide bonding E. Hydrogen bonding

E. Hydrogen bonding

What landmark marks the inferior (aka distal) border of the axillary artery? A. Lateral border of the 1st rib B. Infero-lateral border of pectoralis minor C. Superio-medial border of pectoralis minor D. Superior border of teres major E. Inferior border of teres major

E. Inferior border of teres major

A 28 year old male presents to the ED with a stab wound to the right upper back. The wound appears to lie along the superior edge of the scapula and after careful examination is determined to have traveled straight down to the suprascapular notch. Which structure is likely to be affected as a result of his wound? A. Rhomboid major muscle B. Upper subscapular nerve C. Subscapularis muscle D. Axillary nerve E. Infraspinatus muscle F. Dorsal scapular artery

E. Infraspinatus muscle

The principal type of structure that compartmentalizes the cell is what? A. Glycogen B. Proteins C. DNA D. RNA E. Lipid bilayers

E. Lipid bilayers

During you team's championship football game, your friend and star running back JR got hit very hard in the back on his right side beneath his shoulder pads. While the hit hurt very bad, JR was tough and played through the pain. After winning the game, you noticed JR was not able to raise his right arm above his head while celebrating. In the locker room JR was leaning against the wall with his arms out in front. You walked up to him to congratulate him on the win and noticed he had a "winged scalpula". What nerve did JR damage when he got hit and what are its roots? A. Long Thoracic Nerve, roots C8-T1 B. Lateral Thoracic Nerve, roots C5-C7 C. Thoracodorsal nerve, roots C5-C7 D. Lateral Pectoral Nerve, roots C8-T1 E. Long Thoracic Nerve, roots C5-C7

E. Long Thoracic Nerve, roots C5-C7

The principal locus of ATP production in the cell is which organelle? A. The golgi B. The endoplasmic reticulum C. Lysosomes D. Vacuoles E. Mitochondria F. The nucleus G. Endosomes H. Exosomes

E. Mitochondria

A 19-year-old male presents to your clinic with a deformity of his upper arm. He was lifting weights and heard a "pop". During your examination you notice that he has a mass that has formed on the distal, anterior aspect of his upper arm. What is the name of the nerve that innervates this muscle? A. Median nerve B. Radial nerve C. Axillary nerve D. Ulnar nerve E. Musculocutaneous nerve

E. Musculocutaneous nerve

A woman comes into the clinic presenting with peau'd orange skin on her breast as well as significant breast elevation with pectoral contraction. What is the most appropriate recommendation for surgery? A. Lumpectomy B. Simple mastectomy C. Modified mastectomy D. Total mastectomy E. Radical mastectomy

E. Radical mastectomy

During a bout with a bear, former Soviet wrestler Zangief sustains several debilitating injuries to the right shoulder and humerus while attempting his 360 degree pile-driver, he incorrectly assessed his and the bear's center of gravity. As a result, he lands flat on his right arm and shoulder dorsally and the bear comes down on top of his right axilla ventrally. He sustains lateral fractures of the clavicle and 1st rib. Zangief has paralysis of the anterior compartment of his arm with severe swelling. After your physical examination, you isolate the nerve damage as restricted to the lateral cord of the brachial plexus. What other clinical manifestation would likely be present? (The bear was not harmed in the accident.) A. Motor paralysis to the posterior compartment of the arm. B. Motor paralysis to abduction muscles of the arm. C. Sensory loss to medial cutaneous regions of the arm and forearm with partial motor paralysis of pectoralis major D. Motor paralysis to pectoralis major and pectoralis minor E. Sensory paralysis to the lateral region of the forearm with partial motor paralysis to pectoralis major

E. Sensory paralysis to the lateral region of the forearm with partial motor paralysis to pectoralis major

During a bout with a bear, former Soviet wrestler Zangief sustains several debilitating injuries to the right shoulder and humerus. While attempting his 360 degree pile-driver, he incorrectly assessed his and the bear's center of gravity. As a result, he lands flat on his right arm and shoulder dorsally and the bear comes down on top of his right axilla ventrally. He sustains fractures of the clavicle, 1st rib, and mid humerus. Zangief has paralysis of the anterior compartment of his arm with severe swelling. After your physical examination, you isolate the nerve damage as restricted to the lateral cord of the brachial plexus. What other clinical manifestation would likely be present? A. Motor paralysis to the posterior compartment of the arm B. Motor paralysis to abductor muscles of the arm C. Sensory loss to medial cutaneous regions of the arm and forearm with partial motor paralysis of pectoralis major D. Motor paralysis to pectoralis major and pectoralis minor E. Sensory paralysis to the lateral region of the forearm with partial motor paralysis to pectoralis major

E. Sensory paralysis to the lateral region of the forearm with partial motor paralysis to pectoralis major

Which of the following arteries originates from the distal third of the axillary artery and forms an anastomosis around the scapula? A. Superior Thoracic Artery B. Dorsal Scapular Artery C. Subclavian Artery D. Brachial Artery E. Subscapular Artery

E. Subscapular Artery

Following an accident, a boy has weakness in rotating his arms laterally. Which of the following muscles are affected? A. Teres major and teres minor B. Teres minor and deltoid C. Infraspinatus and deltoid D. Supraspinatus and subscapularis E. Teres minor and infraspinatus

E. Teres minor and infraspinatus

A 9-year-old boy presents to the ER with pain on the right medial elbow accompanied with bruising, swelling and tenderness. He claims to have "broken his funny bone," because of the familiar tingling he is experiencing on his 5th digit and palm. X-ray reveals an avulsion fracture of the right medial epicondyle. Which nerve is most likely damaged? A. Median nerve B. Palmar branch of the median nerve C. Lateral cutaneous nerve of the forearm D. Musculocutaneous nerve E. Ulnar nerve

E. Ulnar nerve

Cindy, a 33-year-old female, was trying to teach her daughter how to doubledutch jump rope since she was a former schoolyard champion of double-dutch back in the day. While jumping, Cindy's foot got wrapped around a rope, and she fell directly onto her right shoulder. In excruciating pain, Cindy, noticed that her arm was medially rotated and it was determined that she acquired an avulsion fracture of the greater tubercle. What is the innervation of the rotator cuff muscle, which is most likely causing the medial rotation of her right arm? A. Subscapularis Nerve B. Medial and Lateral Pectoral Nerve C. Lower Subscapular Nerve D. Upper Subscapular Nerve E. Upper and Lower Subscapular Nerve

E. Upper and Lower Subscapular Nerve

A construction worker slipped off a ladder, fell, and fractured the midshaft of the humerus. What is the most likely presentation of the patient? A. Erb's palsy B. Benedict Hand C. Ape Hand D. Claw Hand E. Wrist Drop

E. Wrist Drop

Match the muscle with the proper nerve of innervation: A. deltoid - thoracodorsal B. teres minor - lower subscapular C. teres major - axillary D. subscapularis - suprascapular E. infraspinatus - suprascapular

E. infraspinatus - suprascapular

A building block of intermediate filaments:

Keratin

A molcular motor that acts on microtubules:

Kinesin

A very frantic 20-year-old woman comes into your clinic for a breast exam. She states that she feels a lump in her right breast and is terrified because her grandmother had breast cancer and she fears she may too. Upon examination you do feel a lump in the inferior medial "quadrant" of the right breast, so you order a mammogram. Based on you extensive knowledge of anatomy, you know that if lump is malignant and metastasizing, to which lymph nodes would it most likely spread to first?

Subdiaphragmatic lymph nodes

Volkmann's contracture ischemia is most associated with tissue damage due to obstruction of what structure?

Occlusion or laceration of brachial artery and the median nerve

The pectoralis major muscle is attached to the ___________ of the humerus.

Intertubercular groove

A protein that binds actin and help to prevent its polymerization:

Profilin

After sustaining a hard fall on the right shoulder, you friend goes to the ER. It is determined that he has a type III shoulder separation. What ligament(s) have been torn in this injury?

Right acromioclavicular, right conoid and right trapezoid

You are out with your special someone, celebrating the 5th year anniversary. You decide to go to a fine french restaurant and are very excited because you've heard great reviews about this especially overpriced restaurant. As you were paying your bill, you noticed a waiter walk by with his arm medially rotated, adducted and extended. You are outraged by this blatant gesture for a tip, and complain to the manager. The manager explains to you apologetically, that the waiter actually has a birth defect, something to do with being pulled out of the womb by the arm. You recollect that he probably has a severed ___________ portion of his brachial plexus roots, resulting in ____________. As you leave the restaurant you hate yourself for being so judgmental. A. Middle (C5-C6); Erb's palsy B. Superior (C4-C5); Klumpke's paralysis C. Inferior(C8-T1); Shoulder Dystocia D. Superior (C5-C6); Erb's Palsy E. Superior(C6-C7); Shoulder Dystocia

D. Superior (C5-C6); Erb's Palsy

A family physician was seeing a patient for an annual check up. She noticed that her patient's shoulder was slightly elevated, so she sent her patient to get a CT scan. The scan showed that the muscle originating from the junction of the first rib and its cartilage and inserting into the groove of the posterior surface of the clavicle was atrophied, with no other signs of damage. The physician knows a possible diagnosis is that the nerve innervating this muscle could be damaged. From which portion of the Brachial Plexus does this nerve originate from? A. Superior Root B. Anterior Division C. Lateral Cord D. Superior Trunk

D. Superior Trunk

The supra scapular nerve is a direct branch of what part of the brachial plexus? A. Lateral Cord B. C7 Nerve Root C. Inferior Trunk D. Superior Trunk

D. Superior Trunk

A 29 year-old housepainter with diagnosed but untreated apiphobia (fear of bees) falls off a ladder when he sees a bee coming towards him. He presents to the ER holding his left upper limb with his other hand. A radiograph confirms a fracture of the middle one third of his clavicle. In what position is the medial clavicular fragment most likely to be displaced? A. Medially B. Laterally C. Inferiorly D. Superiorly E. No displacement

D. Superiorly

A patient is having pain in his shoulder and trouble abducting his left arm. What is the nerve that innervates the muscle responsible for the first 15 degrees of abduction of the arm? A. Axillary nerve B. Spinal accessory nerve C. Dorsal scapular nerve D. Suprascapular nerve

D. Suprascapular nerve

Mrs Jessica Pearson comes into your office complaining of weakness in her left arm, stating that she is unable to initially lift her arm but says "if I help it just get beyond a certain point I have no problem." What is the MOST likely muscle affected? A. Deltoid B. Subscapularis C. Infraspinatus D. Supraspinatus E. Teres minor

D. Supraspinatus

Which of the following is true regarding abduction of the arm? A. Supraspinatus muscle at 15-30 degrees B. Deltoid muscle at 0-15 degrees C. Subscapularis muscle at 0-15 degrees D. Supraspinatus muscle at 0-15 degrees E. Subscapularis muscle at 15-30 degrees

D. Supraspinatus muscle at 0-15 degrees

Ionic bonds occur in proteins where and how often? A. They constitute a major stabilizing force for folding B. They occur often on the surface of proteins C. They occur in all proteins to stabilize their core D. They occur occasionally in the interior of proteins E. Ionic bonds are never found in proteins

D. They occur occasionally in the interior of proteins

A 30-year-old man comes to the ER with a stab wound in his back, and he's unable to raise his right shoulder against resistance. The artery supplying the affected muscle is: A. Internal Thoracic B. Dorsal Scapular C. Suparscapular D. Transverse Cervical E. Ascending Cervical

D. Transverse Cervical

A 30-year-old man injured his left arm in a mixed martial arts fight. Radiographs later revealed a fracture of the medial epicondyle of the humerus. Which of the following nerves is most likely injured as a result of this fracture? A. Musculocutaneous B. Radial C. Median D. Ulnar E. Axillary

D. Ulnar

A golfer comes to your office complaining of elbow pain and of difficulty in some hand movements. You suspect that the patient has medial epicondylitis. Which of the following nerves would most likely be affected by this condition? A. Radial nerve B. Musculocutaneous nerve C. Median nerve D. Ulnar nerve E. Axillary nerve

D. Ulnar nerve

Charlie sustained damage to the musculocutaneous nerve after it has supplied the muscles of the arm. What would Charlie's complaint be after this injury? A. Unable to flex his forearm B. Unable to supinate his forearm C. Unable to adduct his humerus D. Unable to feel on the lateral side of his forearm

D. Unable to feel on the lateral side of his forearm

Which base is found in RNA and not DNA? A. Adenine B. Cytosine C. Thymine D. Uracil E. Guanine

D. Uracil

At a bar fight, a patron was cut by a glass bottle. This resulted in a mid-shaft fracture of the humerus. In addition to the deep artery of the arm being severed (resulting in massive bleeding), the corresponding nerve was also severed. How will the patient's hand present? A. Ape Hand B. Claw Hand C. Hand of Benediction D. Wrist Drop E. Volkmann's Contracture

D. Wrist Drop

What are the various densities that can be seen on x-rays from least dense to most dense? a. Air < Soft tissue < Fat < Calcium < Metal b. Air < Fat < Water < Calcium < Metal c. Air < Soft tissue < Water < Organ < Calcium < Metal d. Air < Fat < Calcium < Water < Bone

b. Air < Fat < Water < Calcium < Metal

What is the proper orientation to a medical image taken in the transverse plane? a. As if we are standing face-to-face with the patient b. As if we are standing face-to-back with the patient c. As if we are standing at the patient's feet, looking toward the head d. As if we are standing on the patient's right side, looking toward the left e. As if we are standing South of the patient, looking North

c. As if we are standing at the patient's feet, looking toward the head

A 43-year-old man presents to his physician with severe right lower quadrant abdominal pain. In working up his condition, what should be the physician's first step? a. Order an immediate CT scan of the abdomen. b. Give the patient morphine to ease the pain then schedule him for a routine right lower quadrant ultrasound c. Perform a history and physical examination. d. Call your attorney because this is going to be a tough patient and you may be in trouble.

c. Perform a history and physical examination.

Which of the following is NOT true regarding CT scans? a. By performing CT scan as contrast is administered, we can see vascular filling defects b. CT scans may be reproduced in any imaging plane or as a 3-dimensional image c. CT scan allows for greater spatial and imaging resolution than x-rays d. A CT scan of the chest gives a lower radiation dose than an x-ray of the chest

d. A CT scan of the chest gives a lower radiation dose than an x-ray of the chest

An elderly gentleman presents to the ER with a wrist drop after sustaining a fall at home. He tells you that he slipped and hit his right arm on the edge of the table. You order an X-ray of the arm and notice a midshaft humeral fracture. You suspect damage to the radial nerve. (a) What vascular structure would you be most concerned may also be damaged? (b) what head of the triceps would most likely have lost innervation?

profunda brachii (deep artery of the arm) medial

A patient presents to the emergency room with a cervical collar (i.e., emergency neck brace) with severe lacerations to her face, neck, and shoulder. The patient had been involved in a car accident after being struck posteriorly causing a "whiplash" effect on the patient's neck. MRI diagnostics show a tear due to hyperextension of the neck. What ligament is most likely damaged in this patient? A. Posterior longitudinal ligament B. Posteroanterior longitudinal ligament C. Lateral longitudinal ligament D. Anterior longitudinal ligament

D. Anterior longitudinal ligament

Lesion to the Dorsal Scapular nerve would lead to: A. Complete loss of scapular movement B. Hypertrophy of the supraspinatus muscle C. Inability to superiorly rotate the scapula D. Diminished movement of the scapula

D. Diminished movement of the scapula

The denticulate ligament connects which two meningeal layers? A. Pia mater to Dura mater B. Pia mater to Arachnoid C. Arachnoid to Dura mater D. Subarachnoid to Dura mater

A. Pia mater to Dura mater

The energy-metabolic role of the TCA cycle is to produce what? A. Reducing equivalents B. CO2 C. Oxaloacetate D. Citrate E. Water

A. Reducing equivalents

Which of the two types of starches will be digested faster, Amylose or amylopectin? A. Amylose B. Amylopectin

B. Amylopectin

A 26 year old mother of a toddler is seen for a physical exam. She could not find a baby sitter, but needs to have the exam today to start her new job at a workplace that has daycare, so brought her child to the visit with her. The internist is 25 minutes late and the child is fussy. To start the interview, the physician asks, "How can I help you today?" This is an example of: A. A close ended question B. An open ended question C. A leading question D. A transitional statement E. An assessment

B. An open ended question

A patient presents with a depressed left shoulder and medially rotated humerus at rest. The patient describes the injury to be from a horseback riding accident that happened previously that day. While falling off the back off the horse, the patient tried to cushion her fall with her abducted and outstretched left arm. Although anterior dislocations are the most common presentation of shoulder dislocations, you suspect an inferior shoulder dislocation secondary to: A. Avulsion of the lesser tubercle of the humerus B. Avulsion of the greater tubercle of the humerus C. Avulsion of the deltoid tuberosity D. A demonic horse

B. Avulsion of the greater tubercle of the humerus

A 14-year-old boy presents in the clinic with a fracture of the proximal portion of his humerus. Which of the following nerves is most likely to be injured? A. Radial nerve B. Axillary nerve C. Musculocutaneous nerve D. Ulnar nerve E. Median nerve

B. Axillary nerve

A 17-year-old high school football player suffered an anterior dislocation of the humerus after attacking an opponent from the rival team. This injury resulted from an instability of his glenohumeral joint. What type of synovial joint is the glenohumeral joint? A. Hinge Joint B. Ball and Socket Joint C. Condyloid Joint D. Pivot Joint E. Saddle Joint

B. Ball and Socket Joint

Your attending physician requests that you collect a venous blood sample from the median cubital vein. "Great!" you say. "Finally something I actually know how to do." You proceed to locate what you are positive is a blood vessel in the roof of the cubital fossa. Sure enough, when you puncture the vessel, you are able to extract blood. You become alarmed, however, when you notice the blood is bright red and that your patient seems to be in a large amount of pain. Oops! You probably just punctured the: A. Median nerve B. Brachial artery C. Biciptal aponeurosis D. Radial artery E. Cephalic vein

B. Brachial artery

Greg, the Gym Rat and Will, the Workout Warrior got into an arm wrestling match, where Greg edged out Will in a close fashion. In the process, Will injured his arm and fell to the ground immediately after the match. He could not flex his arm and also lost sensation on the lateral forearm. MRI and CT scans confirmed that Will has a muscle hypertrophy in the axilla, which have contributed to his injury. A nerve originating from which of the following root(s) is most likely injured? A. C5 B. C5, C6 C. C6, C7, C8 D. C5, C6, C7, C8, T1 E. C8, T1

B. C5, C6

What are the spinal roots of the nerve damaged when a patient presents with a "winged scapula"? A. C4, C5 B. C5, C6, C7 C. C6, C7 D. C6, C7, C8 E. C5, C6

B. C5, C6, C7

What are the nerve roots of the thoracodorsal nerve? A. C5, C6 B. C6, C7, C8 C. C8, T1 D. C5 - T1 E. C5

B. C6, C7, C8

What is the smallest amino acid? A. Lysine B. Glycine C. Alanine D. Serine E. Proline

B. Glycine

What artery does the long thoracic nerve closely parallel to? A. Axillary artery B. Lateral thoracic artery C. Thoracoacromial artery D. Thyrocervical artery E. Thoracodorsal artery

B. Lateral thoracic artery

A 27-year-old patient comes into your clinic with shoulder weakness after being involved in a motor vehicle accident over the weekend. After careful examination you determine he is unable to inferiorly rotate his scapula. What muscle is most likely injured? A. Trapezius B. Levator scapulae C. Serratus posterior superior D. Serratus posterior inferior

B. Levator scapulae

A woman undergoes a mastectomy involving her axillary lymph nodes and in the process a nerve was damaged. She walks into your clinic for a follow up and complains that her scapula is not where it used to be. You examine her and notice she has a winged scapula. What nerve and muscle combination is affected? A. Dorsal scapular nerve- Rhomboid major B. Long thoracic nerve- Serratus Anterior C. Spinal accessory nerve- Trapezius D. Subscapular nerves- Subscapularis

B. Long thoracic nerve- Serratus Anterior

Your patient has a midshaft fracture of the humerus. How will the patient most likely present? A. Weakness in pronating forearm B. Loss of wrist extension C. Inability to oppose thumb D. Inability to abduct digits 2-4

B. Loss of wrist extension

A patient comes to your office complaining of weakness in their left arm. You perform a quick examination and find the patient to have difficulty with flexion and supination at their left elbow. You also note loss of sensation in the left lateral forearm. Damage to which nerve best correlates to your findings? A. Axillary n B. Musculocutaneous n C. Median n D. Radial n E. Ulnar n

B. Musculocutaneous n

Would the system in question 10 be an effective buffer at neutral pH? A. Yes B. No C. I have no clue

B. No

Someone comes into your clinic and they report that they've been stabbed in the quadrangular space. After thanking them for knowing their anatomy, you worry that a branch of which cord in the brachial plexus has been damaged? A. Inferior cord B. Posterior cord C. Medial cord D. Middle cord E. Superior cord F. Lateral cord

B. Posterior cord

A 30-year-old man has diminished sensation on the lateral aspect of his upper left arm following a bad fall down the stairs. Which action is also most likely impaired in this patient on the same side? A. Initiation of abduction of the arm B. Raising arm above head C. Medial rotation of the arm D. Adduction of the arm

B. Raising arm above head

A 45-year-old female presents with large dimples on her breast, after further examination you conclude that breast cancer has invaded her glandular tissue. What caused the dimples in her breast? A. Shortened lactiferous ducts B. Shortening of the suspensory ligaments C. Interference with lymphatic drainage D. Contraction of Pectoralis Major

B. Shortening of the suspensory ligaments

The cephalic vein courses within the __________ groove and pierces the ___________ fascia.

Deltopectotal groove, Clavipectoral fascia

What provides horizontal stability to the spinal cord by anchoring it to the dura mater?

Denticulate ligaments

The rowing motion of a colium is powered by ATP hydrolysis performed by this ATPase:

Dynein

The transport of vesicles and organelles from the tips of nerve cell axons to the cell body is powered by ATP hydrolysis performed by this ATPase:

Dynein

The wriggling motion of sperm tails is powered by ATP hydrolysis performed by this ATPase:

Dynein

What is the correct order of the following steps in protein synthesis? 1. A peptide bond is formed 2. The small ribosomal subunit is loaded with initiation factors, messenger RNA, and initiation aminoacyl-transfer RNA 3. The intact ribosome slides forward three bases to read a new codon 4. The primed small ribosomal subunit binds with the large ribosomal subunit 5. Elongation factors deliver aminoacyl-tRNA to bind to the A site A) 1, 2, 5, 4, 3 B) 2, 3, 4, 5, 1 C) 4, 5, 1, 2, 3 D) 3, 2, 4, 5, 1 E) 2, 4, 5, 1, 3

E) 2, 4, 5, 1, 3

Which of the following is necessary for DNA polymerase to polymerize deoxynucleotides? A) A free 5'-hydroxyl group B) dUTP C) CTP D) NAD+ as a cofactor E) A free 3'-hydroxyl group

E) A free 3'-hydroxyl group

While studying the structure of a small gene that was recently sequenced during the Human Genome Project, an investigator notices that one strand of the DNA molecule contains 20 A's, 25 G's, 30 C's, and 22 T's. How many of each base is found in the complete double-stranded molecule? A) A=40, G=50, C=60, T= 44 B) A=44, G=60, C=50, T= 40 C) A=45, G=45, C=52, T= 52 D) A=50, G=47, C=50, T= 47 E) A=42, G=55, C=55, T= 42

E) A=42, G=55, C=55, T= 42

Which of the following statements concerning plasma membrane is true? A) All G proteins are composed of three sub-units B) The glycocalyx is usually composed of phopholipids C) Ion channel proteins are energy dependent (require ATP) D) Gated channels are always open E) Ankyrin binds to band 3 of the RBC plasma membrane

E) Ankyrin binds to band 3 of the RBC plasma membrane

Ricin interferes with the ability of the ribosome to interact with eEF1 and eEF2. How does it do this? A) By binding to the ribosome thereby interfering with the binding of eEF1 or eEF2. B) By binding to the mRNA and preventing the binding of the tRNA being delivered by the factors. C) By depurinating a guanine in the 18S rRNA thereby inactivating the small ribosomal subunit. D) By preventing the large and small subunit from associating. E) By depurinating an adenine in the 28S rRNA thereby inactivating the large ribosomal subunit.

E) By depurinating an adenine in the 28S rRNA thereby inactivating the large ribosomal subunit.

What is the polymerase that generates the primer in eukaryotes called? A) DNA polymerase δ B) DNA polymerase ε C) DNA polymerase β D) DNA polymerase γ E) DNA polymerase α

E) DNA polymerase α

Site of transcriptional activity: A) Nuclear pore complex B) Nucleolus C) Heterochromatin D) Outer nuclear membrane E) Euchromatin

E) Euchromatin

What is a site of mRNA transcription? A) Ribosome B) Nucleolus C) Heterochromatin D) Smooth ER E) Euchromatin

E) Euchromatin

Which of the following statements is not considered to be evidence that the nucleoplasm is organized into special regions? A) Ribosomes are assembled in the nucleolus B) DNA synthesis takes place at sites called replication factories C) Spliceosomes are responsible for splicing the newly synthesized RNAs into their mature form D) The nuclear matrix helps form chromosome territories E) Importin must bind Ran-GTP before it can release its cargo

E) Importin must bind Ran-GTP before it can release its cargo

During a routine physical examination, you perform the biceps reflex test and discover that it is absent, even though it has been present in the past. What nerve is damaged and what spinal levels are associated with this reflex? A. Musculocutaneous nerve, C4 and C5 B. Median nerve, C5 and C6 C. Radial nerve, C5 and C6 D. Radial nerve, C7 and C8 E. Median nerve, C4 and C5 F. Musculocutaneous nerve, C5 and C6

F. Musculocutaneous nerve, C5 and C6

A weight lifter was working out in the gym when he suddenly felt a severe pain in his right shoulder. After the incident, he could not lift his right arm past 15 degrees and also could not laterally rotate the same arm. The injury to the nerve most likely occurred in which of the following locations of the brachial plexus? A. Anterior Division, Lateral Cord B. Anterior Division, Posterior Cord C. Anterior Division, Medial Cord D. Posterior Division, Lateral Cord E. Posterior Division, Posterior Cord F. Posterior Division, Medial Cord

F. Posterior Division, Medial Cord

The spinal cord receives its blood supply from which of the following? A) Segmental medullary arteries B) Posterior spinal arteries C) Anterior spinal artery D) Lumbar arteries E) Intercostal arteries F) Sacral arteries G) All of the above

G) All of the above

A protein concentrated at the centrosome:

Gamma-tubulin

While boiling water to make a spaghetti dinner, Addison bumped his lateral forearm against a scolding pot. He suffered an excruciatingly painful 2nd degree burn. Which of the main terminal branches of the brachial plexus gave rise to the nerve that is responsible for the sensation Addison is feeling?

Musculocutaneous nerve

A molecular motor that acts on microfilaments:

Myosin

Amoeboid locomotion is powered by ATP hydrolysis performed by this ATPase:

Myosin

The contraction of msucle is powered by ATP hydrolysis performed by this ATPase:

Myosin

The storage of glycogen for future use is an example of which of the following choices? A. Anabolism B. Catabolism

A. Anabolism

In order to perform a lumbar puncture on a patient, what are the layers of tissue that the needle would pass through to get a sample of CSF if the puncture is done exactly in the midline? A. Anterior Longitudinal Ligament, Posterior Longitudinal Ligament, Interspinous Ligament, Dura mater B. Supraspinous Ligament, Interspinous Ligament, Ligamentum Flavum, Arachnoid mater C. Interspinous Ligament, Supraspinous Ligament, Ligamentum Flavum, Pia mater D. Supraspinous Ligament, Ligamentum Flavum, Intervertebral Disc, Dura Mater

A. Anterior Longitudinal Ligament, Posterior Longitudinal Ligament, Interspinous Ligament, Dura mater

A patient presents to your office complaining of excruciating pain in his left shoulder over the past couple days after sustaining a ski accident at Big Bear, California. Upon examination, the shoulder is tender. You ask your patient to abduct his left arm and find that he is only able to initiate abduction to about 15 degrees. As a knowledgable physician, you recall from your first semester anatomy course that the most likely damaged nerve is: A. Axillary nerve B. Suprascapular nerve C. Long thoracic nerve D. Infraspinatus nerve

A. Axillary nerve

DNA differs chemically from RNA how? A. By having a hydrogen instead of a hydroxyl at the ribose 2' position. B. By having a hydroxyl instead of a hydrogen at the ribose 2' position C. By having a hydrogen instead of a hydroxyl at the ribose 3' position. D. By having a hydroxyl instead of a hydrogen at the ribose 3' position E. By having a hydrogen instead of a hydroxyl at the ribose 5' position. F. By having a hydroxyl instead of a hydrogen at the ribose 5' position

A. By having a hydrogen instead of a hydroxyl at the ribose 2' position.

Your patient presents with a herniated disk between C5 and C6. Which nerve is most likely compressed? A. C6 B. C5 C. C7 D. C5 and C6

A. C6

Blood type antigens are an example of what type of molecule? A. Glycolipid B. Glycose-aminoglycan C. Glycerophospholipid D. Nucleic acids. E. Starches

A. Glycolipid

A 23-year old baseball pitcher arrives in your office complaining of inability to lift his arm above his head. You notice when attempting to abduct his right arm, he cannot, yet when you assist him half way, he can complete abduction above his head. Which is the nerve most likely damaged in the patient? A. Suprascapular nerve B. Lower subscapular nerve C. Upper and lower subscapular nerve D. Dorsal scapular nerve

A. Suprascapular nerve

A 66-year-old woman who has been successfully treated for heart attack is discharged home. Two days later, the patient returns to the emergency department (ED) complaining that she has felt scared to be home alone. The visiting nurse that had been requested to go to the patient's home after the patient's hospital discharge never showed up. The patient was told by her health insurance company that they would not pay for home nursing visits based on her diagnosis. Which of the following ACGME competencies is most lacking in this scenario? A. Systems-Based Practice B. Professionalism C. Interpersonal and Communication Skills D. Relationship Centered Care E. Improvement in Practice F. Tenets of Medicine

A. Systems-Based Practice

Detergents are differentiated from other lipids by what particular property? A. The ability to form Micelles. B. The ability to form Bilayers. C. The ability to use them as a car wax. D. Their overall hydrophobicity. E. They are amphiphilic.

A. The ability to form Micelles.

Which of the following statements is true regarding the axillary nerve? A. The axillary nerve is accompanied by the posterior circumflex humeral artery B. The axillary nerve innervates the deltoid and teres major muscles C. The axillary nerve is a branch of the medial cord of the brachial plexus D. The axillary nerve is seen in the triangular interval

A. The axillary nerve is accompanied by the posterior circumflex humeral artery

A patient complains of excruciating lower back pain when trying to stand up from a sitting position. During the patient interview, he admits that he "may have injured his lower back while lifting a fellow amateur wrestler over his head during a body slam maneuver." He did not hear or feel any popping sensation during the accident. You correctly hypothesize the most likely cause of the injury is: A. A sprain of the deep muscles of the back, particularly Erector Spinae, which is innervated (sensory only) by the posterior rami of the associated spinal nerves B. A strain of the deep muscles of the back, particularly Erector Spinae, which is innervated (motor and sensory) by the posterior rami of the associated spinal nerves C. A sprain of the deep muscles of the back, particularly Erector Spinae, which is innervated (motor and sensory) by the posterior root of the associated spinal nerves D. A strain of the deep muscles of the back, particularly Erector Spinae, which is innervated (motor only) by the anterior root of the associated spinal nerves E. A strain of the deep muscles of the back, particularly Erector Spinae, which is innervated (motor only) by the anterior rami of the associated spinal nerves

B. A strain of the deep muscles of the back, particularly Erector Spinae, which is innervated (motor and sensory) by the posterior rami of the associated spinal nerves

A couple of hours after your normal breakfast of oatmeal, you go to McDonalds and order a snack of a supersized Soda (nondiet) and fries and promptly eat them while walking to class. What is the likely state of Phosphofructokinase 1, the main regulatory enzyme in glycolysis after eating your snack? A. Carrying out catalysis at normal speed. B. Activated, permitting glycolysis to proceed apace. C. Inhibited, to permit gluconeogenesis to occur. D. The enzyme is unaffected. E. The enzyme is down-regulated through hormone action.

B. Activated, permitting glycolysis to proceed apace.

Which of the following is considered a protein secondary structure? A. Domain B. Alpha-helix C. Hydrogen bond D. Quaternary structure E. Alanine

B. Alpha-helix

During your Pediatrics clinical rotation, you and your team crowd around a 2-month-old baby. Your attending physician is a little cranky this morning and wants to see if you can remember some general anatomical concepts from your old medical school days. He points to the child and asks when the child learns to hold its head up, what type of spinal curvature is it exhibiting? A. Thoracic lordosis B. Cervical lordosis C. Thoracic kyphosis D. Cervical kyphosis

B. Cervical lordosis

Sandy was looking in the mirror and noticed that her right scapula was displaced laterally. Upon visiting her physician, she was diagnosed with Rhomboid Palsy. Which nerve and nerve roots are most likely damaged? A. Thoracodorsal N. C6-C8 B. Dorsal Scapular N. C5 C. Thoracodorsal N. C6-C7 D. Dorsal Scapular N. C3 E. Posterior Rami of Spinal Nerves

B. Dorsal Scapular N. C5

A gang member was taken to the hospital with a knife located at the medial border of his left scapula. What structure might be damaged? A. Accessory Nerve B. Dorsal Scapular Nerve C. Transverse Cervical Artery D. Subscapular Artery E. Suboccipital Nerve

B. Dorsal Scapular Nerve

The Subdural space is a real space. A. True B. False

B. False

While riding a crowded hospital elevator two doctors discuss details of a patient with advanced breast cancer and seem concerned that the patient appears to be in denial of her illness. Which of the following ACGME competencies is most lacking in this scenario? A. Systems-Based Practice B. Professionalism C. Interpersonal and Communication Skills D. Relationship Centered Care E. Improvement in Practice F. Tenets of Medicine

B. Professionalism

Most, though not all, of the functionality of the cell such as covalent chemistry and movement, is carried out by which type of macromolecule? A. Glycogen B. Proteins C. DNA D. RNA E. Lipid membranes

B. Proteins

An analysis is carried out on a purified nucleotide sample and the following percentage of bases are observed: 20 % A; 30 % G; 35% C; 15% U. What type of sample was isolated? A. DNA B. RNA C. NADH D. Coenzyme A. E. Single-stranded DNA

B. RNA

Energy stores in the body are stored in a chemically _____________ state. A. Oxidized B. Reduced C. Ionized D. Neutral E. Non-polar F. Activated

B. Reduced

As the attending anesthesiologist, you are called to the ER to perform a lumbar puncture on a 45-year old male exhibiting signs of meningitis. As you pass the aspiration needle through the skin, what layers, in order will you pass through to gain access to CSF? A. Skin; fat; muscle; pia; dura; arachnoid B. Skin; fat; muscle; dura; arachnoid C. Skin; fat; muscle; dura; arachnoid; pia D. Skin; fat; muscle; arachnoid; pia

B. Skin; fat; muscle; dura; arachnoid

Which of the following is true concerning a spinal nerve? A) It contains only afferent axons B) It contains only efferent axons C) It contains afferent and efferent axons D) It is also called a dorsal primary rami E) It is part of a central nervous system tract

C) It contains afferent and efferent axons

Cranial nerves III-XII are (i) part of what nervous system and (ii) axon fascicles in the spinal cord are part of what nervous system? A) Central nervous system - peripheral nervous system B) Central nervous system - central nervous system C) Peripheral nervous system - central nervous system D) Peripheral nervous system - peripheral nervous system E) Autonomic nervous system - central nervous system F) Autonomic nervous system - peripheral nervous system

C) Peripheral nervous system - central nervous system

Which one of the following nerve(s) innervates the muscles of the suboccipital triangle? A. C1, a purely sensory nerve B. C1 and C2, purely motor nerves C. C1, a purely motor nerve D. C2, a mixed sensory and motor nerve E. C1 and C2, mixed sensory and motor nerves

C. C1, a purely motor nerve

A coal miner injures his back after a work related accident. An MRI scan reveals that his spinal cord has shifted to the right because the lateral pia mater extensions were torn. Which of the following structures is most likely damaged? A. Filum terminale internum B. Coccygeal ligament C. Denticulate ligament D. Choroid plexus E. Tectorial membrane

C. Denticulate ligament

If your thoracodorsal nerve is injured, which actions are compromised? A. Inferiorly rotates the scapula B. Superiorly rotates scapula C. Extension, adduction, and medial rotation of the humerus D. Flexion, abduction, and lateral rotation of the humerus

C. Extension, adduction, and medial rotation of the humerus

Muscle is a major repository (of the whole body total) for which types of energy storage molecules? A. Fat and protein B. Fat and glycogen C. Glycogen and protein D. Glycogen, fat, and protein.

C. Glycogen and protein

You go to McDonalds and order a snack of a large Soda (nondiet), and a large order of fries and promptly eat them while walking to class. What will happen to your Insulin levels? A. Decrease B. Stay the same C. Increase. D. Turnover will be greatly accelerated. E. Precipitation will cause occlusions in your vasculature

C. Increase.

A diabetic patient visited his physician. The physician interrupted the patient several times during the history. The physician appeared rushed and impatient. During the visit the patient never felt he had the chance to share that he had been having chest pains, and later that week he suffered a heart attack at home. Which of the following ACGME competencies is most lacking in this scenario? A. Systems-Based Practice B. Professionalism C. Interpersonal and Communication Skills D. Improvement in Practice E. Tenets of Medicine

C. Interpersonal and Communication Skills

Which muscle belongs to the group of muscles innervated by dorsal rami of spinal nerves? A. Serratus anterior B. Serratus posterior inferior C. Paraspinous muscles D. Latissimus dorsi E. Rhomboid Major

C. Paraspinous muscles

Proteins are polymers of amino acids conjoined through what type of bond? A. Phosphodiester B. Glycosidic C. Peptide D. Phosphoribosyl E. Ester

C. Peptide

The denticulate ligaments of the spinal cord are made of: A. Dura mater B. Arachnoid mater C. Pia mater D. White matter E. Grey matter

C. Pia mater

A 20 year-old man is stabbed in the upper back on the right side with a knife during a bar fight. Upon presentation in the emergency room, the physician notices that the patient is unable to raise his right shoulder. What nerve was most likely damaged during the altercation? A. Dorsal scapular nerve B. Thoracodorsal nerve C. Spinal accessory nerve D. Suprascapular nerve E. Axillary nerve

C. Spinal accessory nerve

A man got into a bar fight and got stabbed on the right side of his upper back. He was rushed to the emergency room where he received stitches to control bleeding. He comes to see the doctor 3 days later with a drooped shoulder. What nerve was affected to cause a drooped shoulder? A. Dorsal scapular nerve B. Thoracodorsal nerve C. Spinal accessory nerve D. Posterior rami of spinal nerves

C. Spinal accessory nerve

While walking home after a long night of studying a student is attacked from behind. He suffers from multiple injuries including a lesion to the dorsal scapular and spinal accessory nerves. What muscles would affected by this lesion? A. Levator scapulae and erector spinae B. Rhomboid major and latissimus dorsi C. Trapezius and levator scapulae D. Splenius cervicis and trapezius

C. Trapezius and levator scapulae

You are assisting in a childbirth during your clinical rotations. While preparing to give the mother an epidural anesthetic for the pain, the Chief Resident turns to you and asks what structures the epidural space is found between. A. Dura mater & arachnoid mater B. Arachnoid mater & pia mater C. Vertebral bone & dura mater D. Pia mater & spinal cord E. Vertebral body & posterior longitudinal ligament

C. Vertebral bone & dura mater

After an MRI showing an abscess that has spread from its site of origin, it is suspected that Prostate cancer has metastasized through the vertebral column. The attending physician looks toward the resident and asks what veins are especially important for this. What is the correct response? A. Basilic vein B. Cephalic vein C. Transverse cervical vein D. Internal vertebral venous plexus

D. Internal vertebral venous plexus

The erector spinae muscle group belongs to which class of back muscles, and receives innervation from which branches of spinal nerves? A. Extrinsic; anterior rami B. Extrinsic; posterior rami C. Intrinsic; anterior rami D. Intrinsic; posterior rami

D. Intrinsic; posterior rami

Which spinal nerve would be damaged with a herniation between the L2 and L3 vertebrae? A. T12 B. L1 C. L2 D. L3 E. L4 F. L5 G. L6

D. L3

During a practical examination, a 1st semester student sees a cadaver lying on its stomach. A muscle with attachments at the nuchal line, nuchal ligament, C7-T12, and the scapula, is reflected exposing a nerve running the length of the muscle. What is the name of the nerve, and how will a deficit of the muscle present in a clinical situation? A. Spinal accessory nerve; Patient cannot protract shoulders B. Cranial Nerve XI; Patient cannot protract shoulders C. Cranial nerve X; Patient cannot elevate shoulders D. Spinal accessory nerve; Patient cannot depress shoulders E. Cranial nerve X; Patient cannot depress shoulders

D. Spinal accessory nerve; Patient cannot depress shoulders

A patient presents to the emergency department after an attempted suicide. The patient attempted to hang himself, but was found by his wife in time to save his life. What vertebrae is most likely to be fractured and what spinal nerve exits inferior to this vertebrae? A. C1 vertebrae and spinal nerve C1 B. C2 vertebrae and spinal nerve C2 C. C3 vertebrae and spinal nerve C4 D. C1 vertebrae and spinal nerve C2 E. C2 vertebrae and spinal nerve C3

E. C2 vertebrae and spinal nerve C3

A family physician receives data from his HMO that his type 2 diabetic patients have higher average Hgb A1C levels (a measure of diabetic blood sugar control) than the quality benchmark for the group. The physician performs a chart audit of his diabetic patients and realizes that many of them had not received dietary counseling since their initial diagnosis. The physician reviews the medial literature, networks with community nutritionists and formulates a plan to refer his diabetic patients for dietary counseling at least once per year. Which of the following competencies does this scenario best describe? A. Systems-Based Practice B. Professionalism C. Interpersonal and Communication Skills D. Relationship Centered Care E. Improvement in Practice

E. Improvement in Practice

A 2-year-old infant was brought to the emergency department by his parents for high temperature, vomiting, extreme shivering and high pitch moaning cry. The initial diagnosis was Meningitis and a CSF analysis was ordered. A Lumbar puncture was performed where a spinal needle was inserted to draw CSF needed for the analysis. At which vertebral level would the puncture take place? A. T12-L1 B. L1-L2 C. L2-L3 D. L3-L4 E. L4-L5

E. L4-L5

During a motorcycle accident, a 25-year-old male landed on the left lateral side of his rib cage. He was found with his right upper limb abducted. It was then noted "winging" of the right scapula. Which nerve was likely damaged in the accident? A. Spinal accessory nerve B. Lateral pectoral nerve C. Phrenic nerve D. Vagus nerve E. Long thoracic nerve

E. Long thoracic nerve

When you treat fat with lye (a strong base), you make soap. This is because of the properties of fatty acids and monacylglycerols to form what kind of structure? A. Bilayers B. Monolayers C. Liposomes D. Vesicles E. Micelles

E. Micelles

During his first game as running back, Andrew was running with the ball and was tackled from behind. In an effort to break his fall, he outstretched his right arm and felt an immediate snap and then a loss of sensation. Upon returning to the sideline, the trainer had him attempt to abduct his arm against resistance, but he could not initiate the abduction without feeling excruciating pain. Which muscle and nerve has Andrew most likely injured? A. Deltoid - axillary nerve B. Teres major - lower subscapular nerve C. Supraspinatus - upper subscapular nerve D. Deltoid - suprascapular nerve E. Supraspinatus - suprascapular nerve

E. Supraspinatus - suprascapular nerve

The principal difference between starch and glycogen is what? A. The type of glycosidic linkage used. B. Glycogen contains glucose while starch is made of galactose C. Starch contains only (1-4) linkages, while glycogen contains both (1-4) and (1-6). D. Starch is linear while Glycogen is branched. E. The frequency of the branch points.

E. The frequency of the branch points.

Which of the following sugars is not a reducing sugar? A. Glucose B. Fructose C. Galactose D. Mannose E. Maltose F. Lactose G. Sucrose

G. Sucrose

A patient walks into your office with drooped shoulder, you suspect spinal accessory nerve injury. How would you test it?

Shrug shoulders against resistance

A person is stabbed in the back exactly in the midline and the knife goes all the way through to the anterior of the vertebral column. From most superficial to deep, name the ligaments the knife passes through.

Supraspinous, Interspinous, ligamentum flavum, posterior longitudinal ligament, and anterior longitudinal ligament

Which of the following is true regarding x-rays? a. They are produced by a cathode ray tube b. They consist of low-frequency electromagnetic waves c. They depend on the reflective properties of tissue to create images d. They require the administration of a radioactive substance to a patient

a. They are produced by a cathode ray tube

A 26-year-old woman who is 14 weeks pregnant presents to her physician with acute abdominal pain. Her physician suspects acute appendicitis, but is reluctant to expose the fetus to ionizing radiation. Which of the following studies DOES NOT utilize ionizing radiation? a. X-rays (radiographs) b. Magnetic resonance imaging (MRI) c. Computed tomography (CT) d. Nuclear medicine (scintigraphy)

b. Magnetic resonance imaging (MRI)

Which of the following imaging modalities does NOT use x-rays? a. Fluoroscopy b. Magnetic resonance imaging (MRI) c. Mammography d. Computed tomography (CT)

b. Magnetic resonance imaging (MRI)

. Single photon emission computed tomography (SPECT) is utilized in which modality? a. Doppler ultrasound b. Nuclear medicine c. MRI d. Tomosynthesis

b. Nuclear medicine

Which of the following is NOT one of the methods recommended to study for and retain imaging anatomy? a. Try to synthesize imaging anatomy with your cadaver dissection and classroom material. b. Allow at least 24 hours after studying to take the first practice exam. c. Go through the lecture over and over until you are entirely familiar with it. d. Try to draw pictures of anatomy, including imaging anatomy, from memory e. Test yourself and each other.

c. Go through the lecture over and over until you are entirely familiar with it.

. A 43-year-old woman presents to her physician with a 5-day history of low back pain after moving a couch. The physician orders a 5-view lumbar spine x-ray series, but the patient is concerned about the amount of ionizing radiation she will receive. Which of the following is NOT true concerning ionizing radiation? a. It can potentially cause damage to cells leading to birth defects and cancer. b. It occurs at the higher end of the electromagnetic spectrum with higher frequency and shorter waveforms. c. It has enough energy to remove charged particles from atoms, resulting in ions. d. Everyone is at equal risk of undergoing damage from ionizing radiation.

d. Everyone is at equal risk of undergoing damage from ionizing radiation.


Conjuntos de estudio relacionados

Respiratory Centers of the Brain

View Set

CHAPTER 40 - NURSING CARE OF THE CHILD WITH AN ALTERATION IN GAS EXCHANGE/RESPIRATORY DISORDER

View Set

L7 27: Nursing Management: Upper Respiratory Problems

View Set

Disasters and Failures - Final Exam

View Set

Chapter 26. Bipolar and Related Disorders

View Set

Bearing Capacities (Ch16) COMBINED

View Set